Любознательным учащимся

Скачкова Татьяна Геннадьевна

В этом разделе моего сайта вы найдёте много интересных фактов из истории математики, всевозможные математические фокусы, головоломки и интересные задачки. Учитесь мыслить нестандартно! Развивайте логику, смекалку и повышайте эрудицию!

Скачать:


Предварительный просмотр:

Олимпиада по математике Муниципальный этап 2014–2015 уч. г.

Задачи с указаниями и решениями

7 класс

7.1.        Велосипедист планировал доехать из пункта А в пункт В за 5 часов, двигаясь с постоянной скоростью. С намеченной скоростью он двигался до середины пути, а потом решил увеличить скорость на 25%. С новой скоростью он доехал до пункта В. Сколько времени занял весь путь?

Ответ: 4 часа 30 минут. Указание. Пусть а – расстояние между пунктами А и В, v –  запланированная скорость. Тогда . Увеличенная скорость равна 1,25 v. Весь путь велосипедист проедет за  (час).

7.2.        Вася накопил 80 рублей 5-копеечными монетами. Чтобы отдать долг в 25 рублей другу, он стал отсчитывать монеты, но сбился со счета и решил использовать чашечные весы. Как ему выделить нужную сумму за 4 взвешивания, если гирек у него нет?

Указание. Первым взвешиванием Вася делит все монеты на две равные по весу кучки и получает две кучки по 40 рублей. Далее он аналогично делит одну кучку в 40 рублей на две равных. Еще 2 раза проделав такие взвешивания, он в результате будет иметь две кучки по 5 рублей и три кучки в 10, 20 и 40 рублей. Сложив кучки в 5 и 20 рублей, он получит нужную сумму.

7.3.        Агент 007 хочет зашифровать свой номер с помощью двух натуральных чисел т и п так, чтобы . Сможет ли он это сделать?

Ответ: сможет. Указание. .

7.4.        Квадрат разрезали на два прямоугольника. Оказалось, что периметры обоих прямоугольников – целые числа. Можно ли сделать вывод, что периметр исходного квадрата также целое число?

Ответ: нет, нельзя. Указание. Приведем такой пример: квадрат со стороной 5/6 разрежем на два прямоугольника длины 5/6 и ширины 1/6 и 4/6. Тогда периметры прямоугольников равны  и . Комментарий. Прийти к подобному примеру можно так: пусть а – сторона квадрата, х – ширина одного из прямоугольников. Тогда   ширина другого прямоугольника, а их периметры равны  и . Сумма этих периметров равна 6а, значит, а имеет вид , где п – натуральное число. Поэтому  (m – натуральное). При п = 5, т = 2 получается наш пример.

7.5.        Есть п палочек длины 1, 2, 3, …, п (см), из которых надо сложить равносторонний треугольник. Можно ли это сделать, если а) п = 100; б) п = 99? (Палочки ломать нельзя, надо использовать все палочки.)

Ответ: а) нельзя; б) можно. Указание. Очевидно, суммарная длина всех палочек должна делиться на 3, чтобы можно было распределить их на три равные по длине части. а) Но сумма 1+2+…+100 не делится на 3 (чтобы в этом убедиться, не обязательно считать сумму 1+ 2+…+100 = 10150; можно разбить все 100 чисел так: (1+2+3) + (4+5+6) + … + (97+98+99) + 100, и во всех тройках сумма делится на 3, а 100 на 3 не делится). б) Разобьем все 99 чисел на последовательные девятки (1+2+…+9) + (10+11+…+18) + … + (91+92+…+99). Каждую такую девятку вида 9k+1, 9k+2, … 9k+9 разобьем на три группы с одинаковой суммой, а именно: 9k+1 + 9k+5 + 9k+9 = 9k+2 + 9k+6 + 9k+7 = 9k+3 + 9k+4 + 9k+8. Собирая палочки каждой из трех групп для разных k = 0, 1, …, 10, мы получим три одинаковые суммарные длины.

8 класс

8.1.        Велосипедист планировал доехать из пункта А в пункт В за 5 часов, двигаясь с постоянной скоростью. С намеченной скоростью он двигался до середины пути, а потом решил увеличить скорость на 25%. С новой скоростью он доехал до пункта В. Сколько времени занял весь путь?

Ответ: 4 часа 30 минут. Указание. См. задачу 7.1.

8.2.        Агент 007 хочет зашифровать свой номер с помощью двух натуральных чисел т и п так, чтобы . Сможет ли он это сделать?

Ответ: сможет. Указание. См. задачу 7.3.

8.3.        Дан параллелограмм ABCD. Точки M и N – середины сторон ВС и CD соответственно. Найдите отношение площади четырехугольника AMND к площади параллелограмма.

Ответ: . Указание. Пусть . Имеем . Далее,  (т.к. MN – средняя линия в треугольнике BCD), поэтому . Тогда .

8.4.        Имеется п спичек длины 1, 2, 3, …, п (см). Требуется сложить из них равносторонний треугольник. Можно ли это сделать, если а) п = 100; б) п = 99? (Спички ломать нельзя, надо использовать все спички.)

Ответ: а) нельзя; б) можно. Указание. См. задачу 7.5.

8.5.         У Пети есть 4 медных советских монеты – по одной номиналом 1, 2, 3 и 5 копеек. Он узнал такой факт: эти монеты должны весить ровно столько граммов, каков их номинал. Петя хочет проверить этот факт с помощью чашечных весов. Сможет ли он это сделать, если у него есть всего одна гирька в 9 граммов?

Ответ: да, сможет. Указание. Пусть х – вес 1-копеечной монеты, у – вес 2-копеечной монеты. Если проверяемый факт верен, то (х + у) – вес 3-копеечной монеты, а (х + 2у) – вес 5-копеечной монеты. Для того, чтобы в этом убедиться, мы проделаем два взвешивания: 1) 1коп. + 2коп. = 3коп. (?); 2) 2коп. + 3коп. = 5коп. (?). Если хотя бы одно из этих равенств не выполняется (весы не в равновесии), то проверяемый факт неверен. Если равенства выполнились, проверим, проведя еще два взвешивания, следующие равенства: 1коп. + 3коп.+ 5коп. = 9 гр. (?) и 2коп. + 3 коп. + 5коп. = 1коп. + 9 гр. (?). Если хотя бы одно из этих равенств не выполняется, то проверяемый факт неверен. Если же оба равенства выполняются, то имеем два уравнения:  и , из которых получим х =1, у = 2, т.е. проверяемый факт верен.

9 класс

9.1.        Решите уравнение .

Ответ: нет решений. Указание. Заметим, что данное уравнение – квадратное (коэффициенты при х3 уничтожаются). У данного квадратного трехчлена вершина находится в точке с абсциссой ; в этом можно убедиться либо непосредственным подсчетом, либо следующим образом: если ввести замену t = x – 50, то квадратный трехчлен запишется в виде  + , и легко видеть, что коэффициенты при нечетных степенях t уничтожаются, а значение при t = 0 (ордината вершины) положительна, причем старший коэффициент (при t2) также положителен. Значит, уравнение корней не имеет.

9.2.        На доске записано несколько целых чисел. Коля заменил каждое число (стерев его) следующим образом: вместо четного числа он записал его половину, а вместо нечетного – удвоенное. Могло ли оказаться так, что сумма новых чисел и сумма исходных совпали, если сумма исходных чисел равнялась а) 2014; б) 2013?

Ответ: а) не могло; б) могло. Указание. Обозначим через А начальную сумму четных чисел на доске, В – сумму нечетных чисел. Тогда должно выполняться равенство . Значит, сумма на доске должна быть равна А + В = 3В=n. В случае а) при п = 2014 это приводит к противоречию с делимостью на 3. В случае б) при п = 2013 легко проверить такой пример: на доске записаны два числа а = 1342 и b = 671 = a/2.

9.3.        Дан четырехугольник ABCD, в который можно вписать окружность. Докажите, что две окружности, вписанные в треугольники АВС и ADC, касаются диагонали АС в одной и той же точке.

Указание. Пусть AB = a, BC = b, CD = c, AD = d, AC = e и пусть окружность, вписанная в треугольник АВС, касается АС в точке М. Тогда из свойства касательных получим, что . Аналогично, для окружности, вписанной в треугольник ACD и касающейся АС в точке N, получим: . Равенство AM = AN равносильно равенству a – b = d – c  a + c = b + d, а последнее равенство – это известное условие для четырехугольника, в который можно вписать окружность.

9.4.        Два корабля  идут по морю перпендикулярными курсами так, что оба должны пройти через фиксированную точку О (каждый – в свой момент времени. После прохождения точки О корабли, не останавливаясь, продолжат своё прямолинейное движение). Скорости кораблей одинаковы и постоянны. В полдень корабли еще не прошли точку О и находились от нее в 20 км и 15 км соответственно. Будут ли корабли далее в какой-то момент в зоне видимости друг друга, если видимость в этот день 4 км?

Ответ: да, будут. Указание. Пусть a = 20, b = 15, v – скорость кораблей. Тогда расстояние между ними через время t после полудня равно (по теореме Пифагора) . Квадратный трехчлен  с положительным старшим коэффициентoм 2v2 принимает наименьшее значение в вершине . При этом значении  и поэтому расстояние в момент  равно  (легко видеть, что к моменту t0 один из кораблей уже прошел точку О, а другой еще нет).

9.5.         У Пети есть 4 медных советских монеты – по одной номиналом 1, 2, 3 и 5 копеек. Он узнал такой факт: эти монеты должны весить ровно столько граммов, каков их номинал. Петя хочет проверить этот факт с помощью чашечных весов. Сможет ли он это сделать, если у него есть всего одна гирька в 9 граммов?

Ответ: да, сможет. Указание. См. задачу 8.5.

10 класс

10.1.        Решите уравнение .

Ответ: нет решений. Указание. См. задачу 9.1.

10.2.        На доске записано несколько целых чисел. Коля заменил каждое число (стерев его) следующим образом: вместо четного числа он записал его половину, а вместо нечетного –удвоенное. Могло ли оказаться так, что сумма новых чисел и сумма исходных совпали, если сумма исходных чисел равнялась а) 2014; б) 2013?

Ответ: а) не могло; б) могло. Указание : см. задачу 9.2

10.3.        Два корабля  идут по морю перпендикулярными курсами так, что оба должны пройти через фиксированную точку О (каждый – в свой момент времени. После прохождения точки О корабли, не останавливаясь, продолжат своё прямолинейное движение). Скорости кораблей одинаковы и постоянны. В полдень корабли еще не прошли точку О и находились от нее в 20 км и 15 км соответственно. Будут ли корабли далее в какой-то момент в зоне видимости друг друга, если видимость в этот день 4 км?

Ответ: да, будут. Указание. См. задачу 9.4.

10.4.        Дан треугольник АВС. Точка Р – центр вписанной окружности. Найдите угол В, если известно, что , где  – радиусы описанных окружностей треугольников АВС и АРС соответственно.

Ответ: 60°. Указание. Пусть . Тогда , поскольку . Пусть . По известной формуле (для теоремы синусов)  (в треугольнике АВС) и  (в треугольнике АРС). Отсюда получаем уравнение    (т.к. <).

10.5.         а) Докажите неравенство  для всех натуральных чисел п; б) существует ли такое натуральное п, для которого , где [a] означает целую часть числа а?

Ответ: б) не существует. Указание. а) После возведения обеих частей в квадрат и отделения квадратного корня получим равносильное неравенство . б) Предположим, от противного, что такое п существует. Тогда для некоторого натурального k будет выполнено двойное неравенство . Рассмотрим неравенство  . Заметим, что квадраты целых чисел не могут давать остатки 2 и 3 при делении на 9 (можно перебрать все возможные остатки, это числа 0, 1, 4, 7). Значит, неравенство  означает фактически, что . Поэтому получаем , что приводит к противоречию, т.к. на самом деле правая часть меньше левой при всех п (это доказывается точно так же, как в пункте а)).

11 класс

11.1.        Найдите область определения функции .

Ответ: , k – целое. Указание. Выражение в скобках под корнем положительно при всех х (см. решение задачи 9.1). Осталось записать решение неравенства .

11.2.        В пространстве с прямоугольной системой координат дан прямоугольный параллелепипед. Известно, что все его вершины имеют целочисленные координаты, причем разные вершины лежат внутри разных октантов. Ребра параллелепипеда параллельны координатным осям. Найдите диагональ параллелепипеда, если его объем равен 2014. (Октант – это часть пространства, ограниченная тремя координатными плоскостями.)

Ответ: . Указание. Пусть a, b, c – стороны параллелепипеда. Имеем abc = 2014. Заметим, что числа a, b, c – натуральные, как следует из условий задачи (вершины с целочисленными координатами, а ребра параллельны координатным осям). Заметим также, что ни одно из чисел a, b, c не может равняться единице, т.к. вершины лежат внутри разных октантов. Поэтому разложив 2014 на простые множители 2014 = 21953, получим, что a, b, c – это (с точностью до порядка) числа 2, 19, 53. Поэтому диагональ равна .

11.3.        Дан треугольник АВС. Точка Р – центр вписанной окружности. Найдите угол В, если известно, что , где  – радиусы описанных окружностей треугольников АВС и АРС соответственно.

Ответ: 60°. Указание. См. задачу 10.4.

11.4.         а) Докажите неравенство  для всех натуральных чисел п; б) существует ли такое натуральное п, для которого, где [a] означает целую часть числа а?

Ответ: б) не существует. Указание. См. задачу 10.5.

11.5.        Найдите наибольшее и наименьшее значение функции .

Ответ: наименьшее значение равно –1, наибольшее значение равно 24. Указание. Обозначим , . Тогда  =  = . Пусть . Заметим, что у квадратичной функции t(u) координата вершины  и поэтому t(u) монотонно возрастает на промежутке [–1;1], принимая значения от –2 до 4. Далее, функция  =  принимает наименьшее значение равное –1 при t = –1, а на концах отрезка  принимает значения 0 и 24 соответственно, откуда следует результат.



Предварительный просмотр:

Ошибка! Источник ссылки не найден.


Разбор задач первой части заданий

4-5 класс

1

2

3

4

5

В

Б

В

В

Б

Задача №1

В классе в течение недели 20 учеников получили хотя бы одну «пятёрку», 15 учеников получили не менее двух «пятёрок», 13 учеников получили не менее трёх «пятёрок», 8 учеников получили не менее четырёх «пятёрок», и 3 ученика получили не менее пяти «пятёрок». Сколько всего «пятёрок» получили ученики класса в течение недели?

А. 59

Б. Более 59

В. Не менее 59

Г. Определить нельзя

Решение:

Одну «пятёрку» получили 20 – 15 = 5 учеников. Две «пятёрки» получили 15 – 13 = 2 ученика. Три «пятёрки» получили 13 – 8 = 5 учеников. Четыре «пятёрки» получили 8 – 3 = 5 учеников. Пять и больше «пятёрок» получили 3 ученика. Следовательно, количество «пятёрок», полученных учениками класса в течение недели, не менее 5 + 22 + 35 + 45 + 53 = 59.

Ответ. В. Не менее 59.

Комментарий:

Эту задачу в основном все решили правильно. С нею не справились те, кто не понял, что когда в условии стоит «то-то не менее, чем то-то», то ответ тоже скорее всего будет в терминах «то-то не менее, чем то-то». Хотя так бывает и не всегда. Но важно бывает подумать о наименьшем возможном случае: когда по одной пятерке ровно 20, по две пятерки ровно 15 итд. В данной задаче при таком подходе для поиска решения (это, конечно, еще не само решение, потому что нужно уточнить, почему получится именно наименьший возможный ответ) как раз получился бы верный ответ: не менее 59.

Задача №2

По окончании хоккейного турнира две команды-победительницы набрали одинаковое количество очков. Для установления одного победителя было решено, чтобы эти команды провели между собой несколько игр до тех пор, пока одна из команд не одержит 4 победы. Ничьих в этих играх нет. Какое наибольшее количество игр может оказаться необходимым для определения победителя?

А. 6

Б. 7

В. 8

Г. 9

Решение:

Для установления победителя может понадобиться или 4 игры (если все игры выиграет одна из команд), или 5 игр (одна из команд выиграет 4 игры, а вторая одну), или 6 игр (одна из команд выиграет 4 игры, а вторая две), или 7 игр (одна из команд выиграет 4 игры, а вторая три). Наибольшее количество равно 7.

Ответ. Б. 7.

Задача 2.

Комментарий:

Эту задачу в основном все решили правильно. С нею не справились те, кто ставил ответы наугад. Тут можно было ошибиться тем способом, чтобы сразу поставить наибольший из возможных ответов, раз уж спрашивают про что-то наибольшее (такого типа ошибок очень много).

Задача №3

Группа школьников должна подняться с 1-го этажа на 20-й. В лифт могут войти не более 5 человек. Масса каждого школьника меньше 60 кг, но больше 50 кг. Какому из приведенных значений массы может равняться сумма масс всех школьников, если лифт поднял их за 6 раз, а за 5 раз не мог этого сделать?

А. 2 т

Б. 1 т 800 кг

В. 1 т 700 кг

Г. 1 т 200 кг

Решение:

Из условия следует, что школьников не менее 26, но не более 30. Следовательно, масса всех школьников более 5026 = 1 т 300 кг, но менее 6030 = 1 т 800 кг. Поэтому сумма масс всех школьников может равняться 1 т 700 кг.

Ответ. В. 1 т 700 кг.

Комментарий:

Эту задачу тоже в основном все решили верно. Как правило, ошибались тут те, кто чуть-чуть напутал с вычислениями и выбрал вариант Б.

Задача №4

Часы на рис. 1 идут правильно, а на рис. 2 спешат на 5 с за 1 час. Сколько часов прошло с того момента, когда часы на рис. 2 показывали правильное время?

А. 240 ч

Б. 360 ч

В. 420 ч

Г. 840 ч

Решение:

Разница между показаниями часов составляет 35 мин. Часы на рис. 2 спешат на 524 = 120 с = 2 мин в сутки. Следовательно, прошло 35:2 = 17 суток 12 ч или 24∙17 + 12 = 420 ч с того момента, когда они показывали правильное время.

Ответ. В. 420 ч.

Комментарий:

С этой задачей тоже большинство справилось. Неверный вариант тут обычно выбирали те, кто ставил наугад.

Задача №5

На занятие кружка по математике пришло несколько учеников. Во время занятия каждый из них решил 2 задачи из предложенных 5. Известно, что для любых двух кружковцев есть задача, которую один из них решил, а другой нет. Какое наибольшее количество учащихся могло прийти на занятие?

А. 8

Б. 10

В. 11

Г. 12

Решение:

Так как каждый ученик решил 2 задачи, и для любых двух кружковцев есть хотя бы одна задача, которую один из них решил, а другой нет, то задача сводится к нахождению количества различных наборов по 2 задачи из предложенных 5. Это количество равно . Этот результат можно получить следующими рассуждениями. В качестве первой задачи можно взять любую из 5, для любой выбранной первой задачи есть 4 возможности для выбора второй, при этом любые две задачи окажутся выбранными столько раз, сколькими способами их можно переставить, то есть 2 раза. Число 10 и равно искомому количеству учащихся. Для 11 учащихся требования задачи не выполняются (11 > 10).

Ответ. Б. 10.

Комментарий:

С этой задачей справилось большинство. Ошибались тут в основном те, кто выбирал наибольший ответ из предложенных (как было с Задачей 2), т.е. Г. Впрочем, некоторые, похоже, подумали, что «совсем уж наибольший ответ выбирать не стоит» (не всегда эффективная стратегия в тестовых заданиях кстати: работает только примерно в 25% случаев) и выбрали В.


6-7 класс

1

2

3

4

5

А

Г

В

В

Б

Задача №1

В классе 91% учащихся имеет мобильные телефоны, 72% имеют ноутбуки и 43% имеют планшеты. Какой процент учащихся класса заведомо имеет и мобильный телефон, и ноутбук, и планшет?

А. 6%

Б. 7%

В. 8%

Г. 9%

Решение:

Так как 9% учащихся не имеют мобильных телефонов, 28% не имеют ноутбуков и 57% не имеют планшетов, то не более 9 + 28 + 57 = 94% учащихся не имеют хотя бы одно из указанных средств связи. Следовательно, 100 – 94 = 6% учащихся класса  заведомо имеют их все.

Ответ. А. 6%.

Комментарий:

Большинство эту задачу решило. Не решили ее, похоже, те, кто сделал какую-то арифметическую ошибку, либо выбиравшие наугад.

Задача №2

Вчера проверяли настенные часы и будильник и поставили их стрелки правильно. Настенные часы отстают на 2 мин в час. Будильник спешит за час на 1 мин. Когда сегодня утром глянули на часы, то настенные часы показывали 7 ч, а будильник — 8 ч. В котором часу вчера проверяли часы?

А. В 23 ч 40 мин

Б. В 7 ч 40 мин

В. В 13 ч 58 мин

Г. В 11 ч 40 мин

Решение:

За 1 час разница показаний часов увеличивается на 3 мин. Следовательно, прошло 20 ч с момента проверки часов. За 20 ч будильник ушёл вперёд на 20 мин. Следовательно, показания часов сравнивались сегодня в 7 ч 40 мин, а часы устанавливались вчера в 11 ч 40 мин.

Ответ. Г. В 11 ч 40 мин.

Комментарий:

Большинство справилось с этой задачей. Неверный ответ в ней давали те, кто ставил вариант наугад.

Задача №3

На утреннике всем детям поровну раздали 120 конфет. Если бы Петя и Таня не заболели и пришли на утренник, то каждый ребёнок получил бы на 2 конфеты меньше. Сколько детей пришло на утренник?

А. 3

Б. 8

В. 10

Г. 12

Решение:

Так как , то делителями числа 120 являются числа: 1, 2, 3, 4, 5, 6, 8, 10, 12, 15, 20, 24, 30, 60, 120. Количество детей на утреннике может равняться только этим числам. Пары (3; 5), (4; 6), (6; 8), (8; 10), (10; 12), где первое число означает количество детей, пришедших на утренник, а второе — количество детей, которые должны были прийти на утренник, удовлетворяют первому требованию — отличаются на 2. Но второе требование выполняется только для пары (10; 12). Действительно, (120:10) – (120: 12) = 2, а (120:3) – (120: 5)  2, (120:4) – (120:6)  2, (120:6) – (120:8)  2, (120:8) – (120: 10)  2.

Ответ. В. 10.

Комментарий:

С этой задачей в основном все справились.

Задача №4

Три машины выехали одновременно из одного пункта и прибыли в другой пункт одна за другой через равные промежутки времени. Скорость первой из них 80 км/ч, последней — 48 км/ч. Какова скорость машины, пришедшей второй?

А. 66 км/ч

Б. 64 км/ч

В. 60 км/ч

Г. 56 км/ч

Решение:

Обозначим расстояние от старта до финиша через S км, а скорость машины, пришедшей второй, через v км/ч. Тогда  — время движения, соответственно, первой, второй и третьей машин. Из условия следует равенство:

 или . Следовательно, v = 60 км/ч.

Ответ. В. 60 км/ч.

Комментарий:

С этой задачей в основном все справились. Неверный вариант Б, наверно, выбирали те, кто решил, что в ответе должно быть среднее арифметическое скоростей. Но в ответе при честных вычислениях получается так называемое среднее гармоническое: 2/(1/a+1/b) (обратное к среднему арифметическому обратных величин)! Различных видов средних величин бывает еще много, так что считать надо честно!

Задача №5

Средний возраст учащихся класса некоторой школы составляет 11,625 года, а средний рост — 118 см. Какому из приведенных в ответах чисел может равняться количество учащихся в классе, если возраст каждого ученика определяется с точностью до года, а рост — с точностью до 1 см?

А. 16

Б. 24

В. 32

Г. 42

Решение:

Средний возраст учащихся класса равен 11,625 = 11 года. Так как возраст каждого ученика определяется с точностью до года, то количество учащихся кратно 8. Поскольку рост каждого ученика определяется с точностью до 1 см, то количество учащихся кратно 3. Поэтому оно кратно и 3, и 8, то есть кратно 24.

Ответ. Б. 24.

Комментарий:

С этой задачей справились почти все. Неверные ответы давали в основном, похоже, наугад.


8-9 класс

1

2

3

4

5

В

В

Б

Г

В

Задача №1

Компьютерный супервирус повреждает каждую секунду половину объёма информации, содержащейся на диске. На диске было 8000000 байт информации. Через какое наименьшее количество секунд заведомо будет повреждён хотя бы частично файл объёмом 1600 байт?

А. Через 11 с

Б. Через 12 с

В. Через 13 с

Г. Через 14 с

Решение:

Через k секунд остаётся нетронутой  б информации. Указанный файл будет заведомо повреждён, если нетронутой окажется количество информации, удовлетворяющее неравенству  < 1600 или 2k >  = 5 000 (б). Так как 210 = 1024, 211 = 2048, 212 = 4096 < 5000, 213 = 8192 > 5000,  то через 13 с файл будет обязательно повреждён.

Ответ. В. Через 13 с.

Комментарии:

Большинство справилось с этой задачей. Неверный ответ, похоже, давался либо наугад либо при путанице с вычислениями.

Задача №2

Магазин покупает на оптовом складе партию книг в 500 штук по цене 40 зедов  за книгу (зед — условная денежная единица). Увеличение партии на каждые 50 книг приводит к снижению цены одной книги на 2 зеда. Эта скидка сохраняется только в том случае, если общая партия не превосходит 750 книг. Магазин дополнительно заплатил ещё 2100 зедов за книги, купленные сверх 500. На сколько книг увеличилась закупаемая партия?

А. На 50

Б. На 100

В. На 150

Г. В На 200

Решение:

Обозначим через х количество партий по 50 книг, на которые увеличилась закупаемая партия книг. Тогда имеем уравнение: (500 + 50х)(40 – 2х) = 22 100 или х2 – 10х + 21 = 0. Его корни 3 и 7. Корень 7 не удовлетворяет условию задачи, так как в этом случае было бы куплено 500 + 507 = 850 книг, что превышает ограничение в 750 книг. Следовательно, закупаемая партия увеличилась на 503 = 150 книг.

Ответ. В. На 150 книг.

Комментарии:

Тут тоже большинство справились с задачей. Неверный ответ, судя по всему, давался обычно наугад.

Задача №3

В летний спортивный лагерь отправляли детей. В каждом автобусе планировалось поместить одинаковое количество детей. К сожалению, к месту отправления не прибыл один заказанный автобус, поэтому в каждом автобусе пришлось разместить дополнительно по 3 ребёнка. К моменту возвращения за детьми в лагерь прибыло на 2 автобуса больше, чем планировали, и теперь в каждом автобусе ехало на 5 детей меньше, чем предполагали сначала. Сколько детей было отправлено в спортлагерь?

А. 540

Б. 720

В. 810

Г. 900

Решение:

Обозначим через х и у количества заказанных автобусов и количество детей, которых планировали разместить в каждом автобусе. Количество детей, которых отправили в спортлагерь, равно ху.

Это количество также равно (х – 1)(у + 3) или (х + 2)(у – 5). Имеем систему уравнений  или  Решим её:  

Следовательно, было заказано 16 автобусов, в каждом планировалось разместить по 45 детей; всего в спортлагерь отправили 4516 = 720 детей.

Ответ. Б. 720.

Комментарии:

С этим заданием справились почти все. Наиболее распространенный неверный ответ А здесь, похоже, давали из-за ошибки с перемножением.

Задача №4

Два велосипедиста на тренировке движутся равномерно по кольцевой велотрассе в одном направлении. Первый велосипедист проходит трассу на 3 мин быстрее второго и догоняет второго каждые полтора часа. За какое время первый велосипедист проходит трассу?

А. За 18 мин

Б. За 17 мин

В. За 16 мин

Г. За 15 мин

Решение:

Пусть первый велосипедист проходит трассу за х мин, тогда второй — за (х + 3) мин. Если длину круговой трассы обозначить через l км, то их скорости будут соответственно равны  км/мин и  км/мин. Скорость их сближения равна  – . К тому моменту, когда первый велосипедист догоняет второго, он проходит расстояние большее, чем второй, на l км. По условию, имеем уравнение: 90  или х(х + 3) = 3∙90, или х2 + 3х – 270 = 0. Его корни 15 и –18. Условию задачи удовлетворяет только первый корень. Следовательно, первый велосипедист проходит трассу за 15 мин.

Ответ. Г. За 15 мин.

Комментарии:

С этой задачей в основном все справились. Неверные ответы тут явно давались либо наугад, либо по вычислительной ошибке.

Задача №5

В студенческом шахматном турнире приняли участие два школьника. Они вместе набрали 6,5 очков, а все студенты — по одинаковому количеству очков. Сколько студентов участвовало в турнире? В турнире каждый участник играет с каждым по одному разу, за выигрыш даётся 1 очко, за ничью — 0,5 очка, за поражение — 0 очков.

А. 9

Б. 10

В. 11

Г. 12

Решение:

Пусть в турнире участвовало х студентов. Тогда в турнире всего приняло участие (х + 2) человека, и ими было набрано  очков. Действительно, каждый из (х + 2) шахматистов сыграл (х + 1) партию, любая из которых приносит обоим шахматистам вместе 1 очко. Но при этом мы каждую сыгранную партию учитывали дважды: для каждого из двух соперников. Каждый студент набрал  или  очков. Это число будет целым числом или половиной целого числа лишь в том случае, если х = 11, так как  — всегда целое число или половина целого числа.

Ответ. В. 11.

Комментарии:

С этой задачей справилось большинство. Судя по всему, неверный ответ здесь, как правило, давался наугад.




Предварительный просмотр:

Ошибка! Источник ссылки не найден.


Разбор задач третьей части заданий

4-5 класс

Задача №1

Из числа вычли сумму его цифр. Из полученного числа вновь вычли сумму его (полученного числа) цифр, и так делали снова и снова. После четырех таких вычитаний впервые получился нуль. Найти все такие числа. В ответе указать все возможные варианты в порядке возрастания, записав их через запятую. Докажите, что других нет.

Решение

Любое число минус сумма его цифр в десятичной записи делится на 9. Любое число, в котором более одной цифры больше суммы своих цифр. Поэтому перед четвертым вычитание получилось число 9. Число 9 может получиться только из двухзначного числа. Причем это число делится на 9. А если двухзначное число делится на 9, то сумма его цифр тоже 9 или это число 99. Легко заметить, что число 99 не удовлетворяет условию задачи. Поэтому после второго вычитания получилось число 18 (9+9). А после первого – 27 (18+9). Видно, что подходят только числа от 30 до 39. Ответ: все числа от 30 до 39.

Ответ: 30, 31, 32, 33, 34, 35, 36, 37, 38, 39.

Комментарий

Большинство школьников получили верный ответ, но не смогли его полностью обосновать. Чаще всего в качестве обоснования участники рассматривали числа 29 и 40, для которых требовалось меньше 4-х и больше 4-х операций соответственно, откуда делался вывод, что все числа меньше 30 и больше 39 не подходят. Это действительно так, но делать вывод для всех чисел на основании конкретных примеров нельзя, поэтому такие решения засчитывались, как не полностью верные.

Задача №2

Нужно переложить одну спичку так, чтобы равенство стало верным:

Решение

Комментарий

С этой задачей справились практически все школьники.

Задача №3

Четыре спортсмена, Антон, Николай, Алексей и Кирилл, заняли первые четыре места в соревновании по легкой атлетике, причем ни одно место не было разделено между двумя спортсменами.. На вопрос, какое место занял каждый из них, трое болельщиков ответили:

1 – й: «Антон – второе место, Кирилл – третье ».

2 – й: «Антон – первое место, Николай – второе место».

3 – й: «Алексей – второе место, Кирилл – четвертое место».

Оказалось, что каждый болельщик ошибся ровно один раз. Кто занял четвертое место?

Решение

Рассмотрим высказывание 3-го болельщика. Предположим, что Кирилл занял 4-ое место, тогда 2-ое место занял Антон так как, первый болельщик ошибся в том, что Кирилл занял 3-е место. С другой стороны Николай не мог быть 2-ым, значит, из высказывания второго болельщика следует, что Антон занял 1-ое место, но он не может быть одновременно и 1-ым и 2-ым. Значит, Кирилл не занял 4-ое место, отсюда Алексей занял 2-ое место. Значит, Николай не может быть 2-ым, отсюда Антон первый из высказывания 2-го болельщика, значит Антон не занимал 2-го места, отсюда из высказывания первого болельщика Кирилл занял 3-е место. А это значит, что 4-ое место занял Николай.

Ответ: Николай.

Комментарий

Большинство учеников привели полное решение, но встречались работы, в которых приводилось только какие утверждения верные, а какие нет, а вариант, что верными могут быть другие утверждения не рассматривался. В таких работах был получен верный ответ, но доказательство считалось не полным. В этой и подобных задачах необходимо не только приводить ответ, который удовлетворяет всем условиям, но и доказывать, почему никакой другой ответ не подходит.

Задача №4

Четверо владельцев автомобилей решили провести гонки из четырех заездов, при этом меняясь в каждом заезде автомобилями. В результате каждый из владельцев проехал на каждом автомобиле.

  • В первом заезде Дмитрий был на автомобиле Василия, а во втором Василий - на автомобиле Евгения.
  • Николай выиграл третий заезд на своем автомобиле «Москвич», причем он выиграл и все остальные заезды.
  • На «Камазе» во втором заезде ехал Евгений, а в четвертом заезде ехал Дмитрий.
  • В четвертом заезде автомобиль «Камаз» пришел вторым после «Волги».

Кому принадлежит автомобиль «Форд Фокус»?

Решение

Из 4-го условия Николай ехал на Волге в 4-ом заезде и выиграл его, а Дмитрий занял второе место на Камазе. В первых 2х заездах Николай был на Форде и на Камазе, но во втором он не мог ехать на Камазе, так как на нем ехал Евгений, значит в 1-ом заезде Николай ехал на Камазе, а во втором на Форде. Автомобиль Евгения не Камаз и не Форд, так как во втором заезде на автомобиле Евгения ехал Василий, а Евгений и Николай ехали на Камазе и на Форде соответственно. Так же Москвич не является автомобилем Евгения, так как Москвич машина Николая. Значит автомобиль Евгения – Волга. Значит автомобиль Василия либо Камаз либо Форд. Так как Дмитрий ехал в 4-ом заезде на Камазе, а в первом на автомобиле Василия, то автомобиль Василия – не Камаз. Значит автомобиль Василия – Форд.

Ответ: Василию.

Комментарий

Многие участники получили верный ответ, но далеко не все смогли привести полное решение. Часто был пропущен целый кусок обоснования. Скорее всего в уме школьники проделали все необходимые логические шаги, поэтому смогли верно заполнить таблицу, кто и на какой машине ехал в каждом из заездов, но посчитали не нужным приводить в работе длинные пояснения, почему таблица заполнялась именно так и никак иначе. К сожалению, такие решения жюри засчитывало как неполные, потому что в работе должны в письменном виде присутствовать обоснования всех фактов, на основании которых получен ответ, если эти факты не являются общеизвестными.


6-7 класс

Задача №1

Из числа вычли сумму его цифр. Из полученного числа вновь вычли сумму его (полученного числа) цифр, и так делали снова и снова. После четырех таких вычитаний впервые получился нуль. Найти все такие числа. В ответе указать все возможные варианты в порядке возрастания, записав их через запятую. Докажите, что других нет.

Решение

Любое число минус сумма его цифр в десятичной записи делится на 9. Любое число, в котором более одной цифры больше суммы своих цифр. Поэтому перед четвертым вычитание получилось число 9. Число 9 может получиться только из двухзначного числа. Причем это число делится на 9. А если двухзначное число делится на 9, то сумма его цифр тоже 9 или это число 99. Легко заметить, что число 99 не удовлетворяет условию задачи. Поэтому после второго вычитания получилось число 18 (9+9). А после первого – 27 (18+9). Видно, что подходят только числа от 30 до 39. Ответ: все числа от 30 до 39.

Ответ: 30, 31, 32, 33, 34, 35, 36, 37, 38, 39.

Комментарий

Большинство школьников получили верный ответ, но не смогли его полностью обосновать. Чаще всего в качестве обоснования участники рассматривали числа 29 и 40, для которых требовалось меньше 4-х и больше 4-х операций соответственно, откуда делался вывод, что все числа меньше 30 и больше 39 не подходят. Это действительно так, но делать вывод для всех чисел на основании конкретных примеров нельзя, поэтому такие решения засчитывались, как не полностью верные.

Задача №2

Передвиньте одну спичку так, чтобы получилось верное равенство:

Решение

Комментарий

С этой задачей справились практически все школьники.

Задача №3

Даны три сосуда объемом 8, 5, 3 литра. Первый полный, другие пусты. Как разделить воду на две равные части, переливая ее только между имеющимися сосудами?

Решение

Решение показано на схеме:

(8,0,0)(3,5,0)(3,2,3)(6,2,0)(6,0,2)(1,5,2)(1,4,3)(4,4,0).

Комментарий

С этой задачей справились почти все школьники. Некоторые приводили последовательность переливаний, отличающуюся от указанной в решениях:

(8,0,0) -> (5,0,3) -> (5,3,0) -> (2,3,3) -> (2,5,1) -> (7,0,1) -> (7,1,0) -> (4,1,3) -> (4,4,0).

Она длиннее на 1 шаг, но в задаче не требовалось найти оптимальную схему переливаний, поэтому такое решение тоже засчитывалось как полностью верное.

Задача №4

Четверо владельцев автомобилей решили провести гонки из четырех заездов, при этом меняясь в каждом заезде автомобилями. В результате каждый из владельцев проехал на каждом автомобиле.

  • В первом заезде Дмитрий был на автомобиле Василия, а во втором Василий - на автомобиле Евгения.
  • Николай выиграл третий заезд на своем автомобиле «Москвич», причем он выиграл и все остальные заезды.
  • На «Камазе» во втором заезде ехал Евгений, а в четвертом заезде ехал Дмитрий.
  • В четвертом заезде автомобиль «Камаз» пришел вторым после «Волги».

Кому принадлежит автомобиль «Форд Фокус»?

Решение

Из 4-го условия Николай ехал на Волге в 4-ом заезде и выиграл его, а Дмитрий занял второе место на Камазе. В первых 2х заездах Николай был на Форде и на Камазе, но во втором он не мог ехать на Камазе, так как на нем ехал Евгений, значит в 1-ом заезде Николай ехал на Камазе, а во втором на Форде. Автомобиль Евгения не Камаз и не Форд, так как во втором заезде на автомобиле Евгения ехал Василий, а Евгений и Николай ехали на Камазе и на Форде соответственно. Так же Москвич не является автомобилем Евгения, так как Москвич машина Николая. Значит автомобиль Евгения – Волга. Значит автомобиль Василия либо Камаз либо Форд. Так как Дмитрий ехал в 4-ом заезде на Камазе, а в первом на автомобиле Василия, то автомобиль Василия – не Камаз. Значит автомобиль Василия – Форд. Ответ: Форд принадлежал Василию.

Ответ: Василию.

Комментарий

Многие участники получили верный ответ, но далеко не все смогли привести полное решение. Часто был пропущен целый кусок обоснования. Скорее всего в уме школьники проделали все необходимые логические шаги, поэтому смогли верно заполнить таблицу, кто и на какой машине ехал в каждом из заездов, но посчитали не нужным приводить в работе длинные пояснения, почему таблица заполнялась именно так и никак иначе. К сожалению, такие решения жюри засчитывало как неполные, потому что в работе должны в письменном виде присутствовать обоснования всех фактов, на основании которых получен ответ, если эти факты не являются общеизвестными.


8-9 класс

Задача №1

Из числа вычли сумму его цифр. Из полученного числа вновь вычли сумму его (полученного числа) цифр, и так делали снова и снова. После пяти таких вычитаний впервые получился нуль. С какого числа нужно начать? В ответе указать все возможные варианты и доказать, что других нет.

Решение

Любое число минус сумма его цифр в десятичной записи делится на 9. Заметим, что число, с которого мы начинали процесс – двухзначное. А если двухзначное число делится на 9, то сумма его цифр тоже 9 или это число 99. Легко заметить, что число 99 не удовлетворяет условию задачи.. Значит, начиная со второго вычитания число будет каждый раз уменьшаться на 9. Значит перед последним вычитанием было число 9, перед предпоследним 18 и т.д. Следовательно, после первого вычитания получалось число 36. Видно, что подходят только числа от 40 до 49.

Ответ: все числа от 40 до 49.

Комментарий

Большинство школьников получили верный ответ, но не смогли его полностью обосновать. Чаще всего в качестве обоснования участники рассматривали числа 39 и 50, для которых требовалось меньше 5-ти и больше 5-ти операций соответственно, откуда делался вывод, что все числа меньше 49 и больше 49 не подходят. Это действительно так, но делать вывод для всех чисел на основании конкретных примеров нельзя, поэтому такие решения засчитывались, как не полностью верные.

Задача №2

На доске написаны все натуральные числа в ряд, начиная с 1. Какая цифра написана на 107321 месте?

Решение

Однозначных чисел 9. Двузначных 90. Трехзначных 900 и т. д. Заметим, что 1*9+2*90+3*900+4*9000<107321, 1*9+2*90+3*900+4*9000+5*90000>107321. Значит цифра стоящая на этом месте принадлежит пятизначному числу. 1*9+2*90+3*900+4*9000 = 38889. На этом месте стоит цифра 9. Посчитаем, сколько пятизначных чисел было написано до цифры с номером 107321.Это количество равно  . Значит искомая цифра – вторая в 13687ом пятизначном числе.

Ответ: 3.

Комментарий

Многие школьники в этой задаче указывали только ответ, без каких-либо доказательств. Скорее всего, они проделали в уме все необходимые расчеты, но посчитали не нужным приводить их в работе, такие решения считались не верными, тем более, что в данном случае ответ выбирался всего из 10 возможных цифр, поэтому его можно было просто угадать. Кроме того, часто встречалась ошибка, когда школьники писали, что искомой является вторая цифра в числе 13687, на самом деле это вторая цифра числа 23686, то есть 13687-го из пятизначных, хотя ответ и получался таким же, решение оценивалось как не полностью верное.

Задача №3

Даны три сосуда объемом 8, 5, 3 литра. Первый полный, другие пусты. Как разделить воду на две равные части, переливая ее только между имеющимися сосудами?

Решение

Решение показано на схеме:

(8,0,0)(3,5,0)(3,2,3)(6,2,0)(6,0,2)(1,5,2)(1,4,3)(4,4,0).

Комментарий

С этой задачей справились почти все школьники. Некоторые приводили последовательность переливаний, отличающуюся от указанной в решениях:

(8,0,0) -> (5,0,3) -> (5,3,0) -> (2,3,3) -> (2,5,1) -> (7,0,1) -> (7,1,0) -> (4,1,3) -> (4,4,0).

Она длиннее на 1 шаг, но в задаче не требовалось найти оптимальную схему переливаний, поэтому такое решение тоже засчитывалось как полностью верное.

Задача №4

В космический полет набрали команду из четырех ученых: физик, химик, биолог и математик. Национальности их были различные, и, хотя каждый из ученых владел двумя языками из четырех (русский, английский, французский, итальянский), не было такого языка, на котором они могли бы разговаривать вчетвером.

Был язык, на котором могли разговаривать сразу трое. Никто из ученых не владел французским и русскими языками одновременно. Не было языка, на котором могли говорить и биолог, и химик. Но физик, хотя и не знал английского, мог быть для них переводчиком, если биолог и химик захотели бы поговорить друг с другом.  Химик говорил по-русски и мог говорить с математиком, хотя тот не знал ни одного русского слова. Физик, биолог и математик не могли беседовать втроем на одном языке.

На каком языке могли разговаривать трое ученых? Свои рассуждения объяснить.

Решение

По условию ясно, что Физик не говорит на английском, химик говорит на русском, биолог и математик на русском не говорят. Так как химик говорит на русском, то он не может говорить на французском.  Биолог говорит на французском, иначе неизбежно у них с химиком будет общий язык. Пусть теперь Физик говорит на русском. Из того, что французским и русским не владеет одновременно никто, Физик должен говорить на итальянском. Тогда, что бы он мог переводить химику и биологу, биолог должен говорить на итальянском. Тогда математик говорить на итальянском не может и ему остаются только французский и английский. А химик Говорит еще и на английском, чтобы не пересекаться с биологом. Но тогда нет ни одного языка, на котором говорили бы трое. Следовательно, физик говорит на французском. Тогда математик говорит на испанском и английском так как не может говорить на французском. А химик должен говорить на итальянском, что бы физик мог переводить ему и биологу. Тогда искомый язык - итальянский.

Комментарий

Большинство получили верный ответ в задаче, но лишь не многие привели доказательство, как этот ответ был получен. Часто был пропущен целый кусок обоснования. Скорее всего в уме школьники проделали все необходимые логические шаги, поэтому смогли верно заполнить таблицу, кто и на каком языке разговаривал, но посчитали не нужным приводить в работе длинные пояснения, почему таблица заполнялась именно так и никак иначе. К сожалению, такие решения жюри засчитывало как неполные, потому что в работе должны в письменном виде присутствовать обоснования всех фактов, на основании которых получен ответ, если эти факты не являются общеизвестными.


Конкурс для учителей

Задача №1

Из числа вычли сумму его цифр. Из полученного числа вновь вычли сумму его (полученного числа) цифр, и так делали снова и снова. После четырех таких вычитаний впервые получился нуль. Найти все такие числа. В ответе указать все возможные варианты в порядке возрастания, записав их через запятую. Докажите, что других нет.

Решение

Любое число минус сумма его цифр в десятичной записи делится на 9. Любое число, в котором более одной цифры больше суммы своих цифр. Поэтому перед четвертым вычитание получилось число 9. Число 9 может получиться только из двухзначного числа. Причем это число делится на 9. А если двухзначное число делится на 9, то сумма его цифр тоже 9 или это число 99. Легко заметить, что число 99 не удовлетворяет условию задачи. Поэтому после второго вычитания получилось число 18 (9+9). А после первого – 27 (18+9). Видно, что подходят только числа от 30 до 39.

Ответ: все числа от 30 до 39.

Комментарий

Во многих работах был получен верный ответ, но не все смогли его обосновать. Чаще всего в качестве обоснования участники рассматривали числа 29 и 40, для которых требовалось меньше 4-х и больше 4-х операций соответственно, откуда делался вывод, что все числа меньше 29 и 50 не подходят. Это действительно так, но делать вывод для всех чисел на основании конкретных примеров нельзя, поэтому такие решения засчитывались, как не полностью верные.

В некоторых работах терялся один из ответов (обычно 30), либо получался ответ 36 (совсем не обязательно, что исходное число делилось на 9, число становилось кратным 9 уже после первого вычитания).

Задача №2

На столе лежат 15 спичек. Двое игроков берут по очереди 1, 3 или 4 спички. Выигрывает тот, кто забирает последнюю спичку. Кто выиграет при правильной игре? И как надо играть?

Решение

Выигрышная стратегия есть у первого игрока. Своим первым ходом он берет одну спичку. Осталось 14. После хода второго может остаться: 13 (тогда первый берет 4), 11 (тогда первый берет 4), 10 (тогда первый берет 1). У второго 7 или 9 спичек.

Пусть осталось 9. После хода второго может остаться 8, 6, 5 спичек. Тогда первый оставляет 7 или 2.

Пусть осталось 7 спичек. После хода второго может остаться 6, 4, 3 спички. Тогда первый забирает все (если их было 3 или 4) или оставляет 2.

Итак, второго две спички и он может забрать только одну. Одна остается и первый выиграл.

Комментарий

Напомним, что победителем при правильной игре будет тот, и только тот игрок, у которого есть выигрышная стратегия все зависимости от ходов другого игрока. В этой задаче некоторые участники ограничились только ответом, не приведя стратегии, как нужно играть, а это и есть самое главное в решении.

Некоторые участники в своих работах расписывали, какое количество спичек, оставшееся игроку после хода противника является выигрышной или проигрышной позицией. В этих решениях проводился анализ с конца. Напомним, что позиция называется выигрышной, если есть возможность сделать такой ход, чтобы противнику досталась проигрышная позиция. Проигрышной же называется позиция, при которой любой ход приводит противника в выигрышную позицию.

Будем двигаться с конца и обозначим каждую позицию, выигрышная она или проигрышная.

0

1

2

3

4

5

6

7

8

9

10

11

12

13

14

15

п

в

п

в

в

в

в

п

в

в

в

в

п

в

п

в

Получаем, что изначально 1-й игрок находится в выигрышной позиции, значит он сможет выиграть вне зависимости от игры второго.

Задача №3

Даны три сосуда объемом 8, 5, 3 литра. Первый полный, другие пусты. Как разделить воду на две равные части, переливая ее только между имеющимися сосудами?

Решение

Решение показано на схеме:

(8,0,0)(3,5,0)(3,2,3)(6,2,0)(6,0,2)(1,5,2)(1,4,3)(4,4,0).

Комментарий

Некоторые приводили последовательность переливаний, отличающуюся от указанной в решениях:

(8,0,0) -> (5,0,3) -> (5,3,0) -> (2,3,3) -> (2,5,1) -> (7,0,1) -> (7,1,0) -> (4,1,3) -> (4,4,0).

Она длиннее на 1 шаг, но в задаче не требовалось найти оптимальную схему переливаний, поэтому такое решение тоже верное.

Задача №4

В очереди за билетами в кино стоят Юра, Миша, Володя, Саша и Олег. Известно, что:

  • Юра купит билет раньше, чем Миша, но позже Олега;
  • Володя и Олег не стоят рядом;
  • Саша не находится рядом ни с Олегом, ни с Юрой, ни с Володей.

Кто стоит третьим?

Решение

Из условия ясно, что Олег стоит раньше Юры, а Юра раньше Миши. Саша может стоять только рядом с Мишей. Если Саша стоит не последним, то он стоит перед Мишей, а следовательно перед ним может стоять или Юра или Володя, что противоречит условию. Так же Володя не может стоять за Сашей. Володя и Олег не стоят рядом. Следовательно. Володя стоит за Юрой. Таким образом, третьим стоит Володя.

Ответ: Володя.

Комментарий

Многие привели верные решения этой задачи, но в некоторых работах был указан только ответ, либо написаны все номера мальчиков в очереди, в данной задаче нужно было так же объяснить, почему такая расстановка в очереди единственная удовлетворяет всем условиям задачи.


 


Предварительный просмотр:


Предварительный просмотр:

Ошибка! Источник ссылки не найден.


Разбор задач первой части заданий

4-5 класс

1

2

3

4

5

Г

Б

А

Г

Г

Комментарий

Так как в задачах из этой части нужно было только указать ответ без пояснений, мы не можем отследить, как был получен тот или иной неверный ответ. Скорее всего часто школьники указывали букву наугад, когда не могли решить задачу. Тем не менее, в некоторых задачах из всех неверных вариантов предпочитался один. С большинством задач многие школьники справились. Разберем те задачи, в которых доля неверных ответов была значительной.

Задача №1

Вместо звездочек вставьте цифры так, чтобы получилось верное выражение. В ответе укажите цифру, которая стоит между цифрами 7 и 8 в нижней строке.

2   *

×

*   2

*   8

 7   *

7    *     8

А. 3

Б. 4

В. 2

Г. 6

Д. 5

Решение

Цифра, которая стоит вместо * в первом множителе при умножении на 2 должна заканчиваться на 8. Поэтому это 4 или 9. Предположим это 9. Тогда число 29 умножается на какую-то цифру и получается двухзначное число, которое начинается на 7, т.е. находится в диапазоне от 70 до 79. Если 29 умножить на два, то получится число 58, которое меньше 70, а если 29 умножить на 3, то получиться число больше чем 79, поэтому такой цифры нет. Поэтому первый множитель 24. В 4-ой строке стоит число от 70 до 79, поэтому 24, умноженное на какую-то цифру должно быть от 70 до 79. 24*2<70, 24*3=72, 24*4>79, значит второе число 32. Тогда 24*32=768.

Ответ: 6.

Задача №2

Сколько трехзначных чисел можно составить из двух цифр 0 и 7?

А. 2

Б. 4

В. 6

Г. 8

Д. 10

Решение

Заметим, что на первом месте не может стоять 0. Тогда на первом месте по условию задачи обязана стоять цифра 7. На любом другом месте может стоять как цифра 0 так и цифра 7. Каждому варианту количества десятков соответствует два варианта выбора количества единиц, таким образом, всего можно составить 2*2=4 различных трехзначных числа.

Ответ: 4.

Задача №3

В футбольной команде 11 человек, сколькими способами можно выбрать капитана и его заместителя?

А. 110

Б. 100

В. 90

Г. 75

Д. 55

Решение

Пусть капитан выбирается первым. Выбрать капитана из 11 человек можно 11 способами. Из оставшихся человек выбрать заместителя можно 10 способами. Так как каждому способу выбрать капитана соответствует 10 способов выбора заместителя, то 11 надо умножить на 10. Отсюда 11*10=110 способов.

Ответ: 110.

Комментарий

В работах часто встречался вариант Д (55). Если бы из 11 человек мы выбирали двух напарников, этот ответ был бы верным. Действительно, первого напарника мы выбираем 11 способами, а второго 10. Нам нужно умножить 11 на 10, чтобы получить все возможные комбинации, а затем поделить на 2, потому что каждую пару мы посчитали 2 раза. В предложенной же задаче не нужно было делить на 2, потому что здесь выбор пары «капитан Вася и заместитель Петя» (назовем их так) не идентичен выбору пары «капитан Петя и заместитель Вася», это 2 разных способа, поэтому верный ответ 110, а не 55.

Задача №4

На рисунке схема, где прямоугольниками обозначены города, а пунктирными линиями дороги. Рабочим нужно закрыть для ремонта несколько дорог. Какое максимальное количество дорог можно закрыть одновременно, чтобы можно было проехать из любого города в любой?

А. 8

Б. 7

В. 4

Г. 6

Д. 5

Решение

Покажем, что семь дорог нельзя перекрыть. Всего дорог 17, если семь из них перекрыть, то останется всего 10 дорог. А 10 дорог может соединить только 11 городов: первая дорога соединяет два города, а каждая следующая дорога присоединяет только один город. А у нас городов 10. Таким образом, получаем, что семь дорог перекрыть нельзя.

Шесть дорог можно закрыть, например, так:

Ответ: 6.

Задача №5

В наборе из нескольких монет одна фальшивая, поэтому весит меньше, чем другие. Известно, что ее можно отыскать без гирь за два взвешивания на чашечных весах. Каково максимальное число монет в этом наборе?

Замечание. Взвешивание на чашечных весах двух кучек позволяет определить, какая из них весит больше, или установить равенство их веса.

А. 4

Б. 6

В. 8

Г. 9

Д. 10

Решение

Понятно, что при любом взвешивании, мы будем на чаши весов класть по одинаковому количеству монет, иначе мы не получим никакой полезной информации после взвешивания. Любое взвешивание разбивает множество монет на три множества ( монеты на левой и на правой чашах и те которые не участвовали при взвешивании ). И после любого взвешивания однозначно определяется множество, содержащее фальшивую монету: если весы в равновесии, то фальшивая монета не принимала участия во взвешивании. Пусть есть хотя бы 10 монет, тогда после первого взвешивания может остаться множество монет, содержащее фальшивую монету и состоящее не менее чем из 4-х монет. После второго взвешивания может остаться множество с фальшивой монетой, состоящее хотя бы из 2-х монет. Значит не всегда можно определить фальшивую монету за 2 взвешивания среди хотя бы 10-ти монет. Если монет 9, то делим монеты на три равные части. И после первого взвешивания останется 3 подозрительные монеты. Две из них взвешиваем и однозначно определяем фальшивую монету.

Ответ: 9 монет.

Комментарий

Эта задача оказалась самой сложной в работе. С ней справилось меньше половины участников. Чаще всего указывали варианты А (4) и Б (6). По всей видимости школьники смогли отыскать способ найти фальшивую монету из 4 либо 6 монет и подумали, что раз им не удалось придумать для большего числа, то это сделать невозможно. На самом деле это не так. В таких задачах всегда следует придумывать обоснование, почему для большего числа невозможно выполнить условие.


6-7 класс

1

2

3

4

5

Б

Б

Г

Г

В

Комментарий

Так как в задачах из этой части нужно было только указать ответ без пояснений, мы не можем отследить, как был получен тот или иной неверный ответ. Скорее всего часто школьники указывали букву наугад, когда не могли решить задачу. Тем не менее, в некоторых задачах из всех неверных вариантов предпочитался один. С большинством задач многие школьники справились. Разберем те задачи, в которых доля неверных ответов была значительной.

Задача №1

Вместо звездочек вставьте цифры так, чтобы получилось верное выражение. В ответе укажите цифру, заключенную в скобки.

                   

   ×

             8     

             

         

           ()

А. 7

Б. 8

В. 9

Г. 0

Д. 6

Решение

Из условия видно, что если первый множитель умножить на 8, то получится двухзначное число, а если умножить на *, то трехзначное. Значит эта * больше 8 и это цифра, значит это 9. Первый множитель не больше 12, так как 12*8=96, а 13*8=104 - трехзначное. Осталось перебрать числа 10, 11, 12. В произведении должно получиться четырехзначное число. В итоге 12*89=1068.

Ответ: 8.

Задача №2

Сколько четырехзначных чисел, можно составить из двух цифр 0 и 7?

А. 6

Б. 8

В. 10

Г. 12

Д. 16

Решение

Заметим, что на первом месте не может стоять 0. Тогда на первом месте по условию задачи обязана стоять цифра 7. На любом другом месте может стоять как цифра 0 так и цифра 7. Следовательно, вариантов поставить число на одну позицию два. Таким образом, вариантов расставить два числа на трёх позициях 2*2*2=8.

Ответ: 8.

Комментарий

Среди неправильных чаще всего встречались варианты ответов А и Д. Заметим, что вариант А можно было получить, если решать задачу перебором, но при этом потерять 2 допустимых варианта. А вот ответ Д получался, если забыть, что четырехзначные числа не могут начинаться с цифры 0. Если бы нужно было найти количество последовательностей из 4-х цифр, состоящих из 0 и 7, то ответ действительно был бы 16.

Задача №3

Сколько различных слов (необязательно осмысленных) можно получить перестановками букв в слове ИНФОРМАТИКА.

А. 39916800

Б. 19958400

В. 3326400

Г. 9979200

Д. 4989600

Решение

Составим новое слово из букв ИНФОРМАТИКА. У нас есть 11 вариантов выбрать букву на первую позицию. Так как одну и ту же букву мы не можем использовать два раза, то вариантов выбрать букву на вторую позицию 10. Тогда заполнить все позиции возможно  способами. Очевидно, что от перестановок одинаковых букв слово не изменится. Такие буквы И и А встречаются по два раза каждая. Существуют только два слова одинаковые во всем, кроме порядка букв И или А в слове т. е. каждое слово посчитано 4 раза Следовательно ответ .

Ответ: 9979200.

Комментарий

Эта задача оказалась самой сложной в тестовом блоке. Больше половины школьников решили ее неверно. Самым популярным ответом был А, он был бы верным в том случае, если бы в исходном слове все буквы были бы различными, но так как в слове ИНФОРМАТИКА 2 раза встречается буква А и два раза буква И, то получается, что мы по 4 раза посчитали каждое слово, поэтому и ответ в 4 раза меньше.

Задача №4

На рисунке схема, где прямоугольниками обозначены города, а пунктирными линиями дороги. Рабочим нужно закрыть для ремонта несколько дорог. Какое максимальное количество дорог можно закрыть одновременно, чтобы можно было проехать из любого города в любой?

А. 8

Б. 7

В. 4

Г. 6

Д. 5

Решение

Покажем, что семь дорог нельзя перекрыть. Всего дорог 17, если семь из них перекрыть, то останется всего 10 дорог. А 10 дорог может соединить только 11 городов: первая дорога соединяет два города, а каждая следующая дорога присоединяет только один город. А у нас городов 10. Таким образом, получаем, что семь дорог перекрыть нельзя.

Шесть дорог можно закрыть, например, так:

Ответ: 6.

Задача №5

В наборе из нескольких монет одна фальшивая, поэтому весит меньше, чем другие. Известно, что за три взвешивания на чашечных весах без гирь ее можно отыскать. Каково максимальное число монет в этом наборе?

Замечание. Взвешивание на чашечных весах двух кучек позволяет определить, какая из них весит больше, или установить равенство их веса.

А. 6

Б. 9

В. 27

Г. 81

Д. 16

Решение

Понятно, что при любом взвешивании, мы будем на чаши весов класть по одинаковому количеству монет, иначе мы не получим никакой полезной информации после взвешивания. Любое взвешивание разбивает множество монет на 3 множества (монеты на левой и на правой чашах и те которые не участвовали при взвешивании). И после любого взвешивания однозначно определяется множество, содержащее фальшивую монету. Пусть есть хотя бы 28 монет, тогда после первого взвешивания может остаться множество монет, содержащее фальшивую и состоящее не менее чем из 10-ти монет. После второго взвешивания может остаться множество с фальшивой монетой состоящее хотя бы из 4-х монет. После третьего взвешивания может остаться множество с фальшивой монетой состоящее хотя бы из 2-х монет. Значит не всегда можно определить фальшивую монету за 3 взвешивания среди хотя бы 28-ти монет. Если монет 27, то после первого взвешивания останется 9 подозрительных монеты, после второго останется 3 подозрительных монеты, а после 3-го ровно одна.

Ответ: 27 монет.

Комментарий

В этой задаче количество верных и неверных ответов было примерно одинаковым. Самым популярным среди неверных ответов был Б. По всей видимости, школьникам удалось за 3 взвешивания определить фальшивую монету среди 9 монет, а для большего числа не получилось, поэтому и был выбран такой ответ. Заметим, что в кучке из 9 монет фальшивую монету, которая легче остальных можно определить всего за 2 взвешивания, а вот для любого количества монет от 10 до 27 понадобится уже целых 3 взвешивания.


8-9 класс

1

2

3

4

5

Г

Г

Б

Г

Комментарий

Так как в задачах из этой части нужно было только указать ответ без пояснений, мы не можем отследить, как был получен тот или иной неверный ответ. Скорее всего часто школьники указывали букву наугад, когда не могли решить задачу. Тем не менее, в некоторых задачах из всех неверных вариантов предпочитался один. С большинством задач многие школьники справились. Разберем те задачи, в которых доля неверных ответов была значительной.

Задача №1

В примере все цифры заменены буквами, одинаковые – одинаковыми, разные – разными.

РОМА + РОМА + РОМА + РОМА = МАРТ

Найдите букву Т.

А. 0

Б. 2

В. 4

Г. 6

Д. 8

Решение

4*РОМА=МАРТ. Значит 1000<РОМА<2500 => 04000 => М>3.Если 4А<10, то остаток от деления на 10 числа 4М равен Р. Р=1 или 2. Подставляя все цифры от 4 до 9 вместо М видно, что подходит только М=8. Если 9<4А<20, то по аналогичным рассуждениям получаем, что М=5. Если 19<4А<37, то получаем М=5 и М=7. Значит М=5,7,8.Если М=8, то 4А<10 (А=0,1,2) , а значит Р=2 => так как РОМА<2500, то О<5, и остаток от деления числа 4*О+3 на 10 равен А. Подставляя вместо О цифры от 0 до 4,видно, что подходит только число 2 => О=2, но тогда 4*О+3>10, а значит М=9, противоречие. Если М=7, то 4А>29 (А=8,9) и Р=1, тогда 4*О+3>29 и 4*О+3 дает остаток 7 при делении на 10, что невозможно, противоречие. Значит М=5. Отсюда Р=1, 9<4*О+2<20, значит О=2,3,4. 9<4А<20 отсюда А=3,4. 4*О+2 четно, значит не может давать нечетный остаток при делении на 10, значит А=4, значит 4*О+2=14, отсюда О=3. 1354*4=5416. Значит Т=6.

Ответ: 6.

Комментарий

Все неправильные ответы в этой задаче встречались в работах примерно одинаковое количество раз, по всей видимости школьники просто пытались угадать верный ответ.

Задача №2

Сколько пятизначных чисел можно составить из трёх цифр 0, 1 и 7?

А. 48

Б. 54

В. 81

Г. 162

Д. 243

Решение

Заметим, что на первой позиции не может стоять 0. Тогда выбрать цифру для первой позиции мы можем двумя способами. На любую другую позицию мы можем поставить любую из трёх цифр, следовательно всего вариантов расстановки цифр 2*3*3*3*3=162

Ответ: 162.

Комментарий

С этой задачей так же справилось большинство учеников. Заметим, что в отличие от 6-7 класса практически все школьники понимают разницу между пятизначным числом (которое не может начинаться с 0) и последовательностью из пяти цифр (которая начинаться с 0 может), поэтому на вариант Д мало кто купился.

Задача №3

Какое минимальное количество карт из колоды (36 карт) нужно вытащить, чтобы гарантированно найти все 4 масти?

А. 4

Б. 28

В. 36

Г. 18

Д. 9

Решение

Карт каждой масти 9. Если вытащить 27 карт, то все они могут оказаться только трех мастей. Пусть вытащили 28 карт и среди первых 27 карты только трех мастей, тогда в колоде останутся карты только четвертой масти. И 28-я карта будет уже 4-й масти.

Ответ: 28.

Комментарий

В этой задаче среди неправильных самым популярным ответом был вариант Д, что странно, потому что вытащив 9 карт, в них может не оказаться даже 2х карт разных мастей, а не то, что 4х, ведь все они могут быть одной масти, хотя вероятность вытащить все карты одной масти из колоды очень маленькая.

Задача №4

1000 команд играют на «вылет» в турнире по волейболу. Какое минимальное количество матчей необходимо для нахождения победителя?

А. 10

Б. 499

В. 500

Г. 999

Д. 1000

Решение

За один матч отсеивается ровно одна команда. В итоге осталась только одна команда. Следовательно, было отсеяно ровно 999 команд. Значит, потребуется 999 матчей.

Ответ: 999.

Комментарий

В этой задаче многие участники выбирали вариант А. По всей видимости, они не разобрались, что такое турнир на вылет. В турнире на вылет в каждом матче играют две команды, после чего победитель остается в турнире, а проигравший выбывает. Заметим, что если бы в турнире были раунды, в которых одновременно играют все команды друг с другом (например в первом раунде играют 500 пар друг с другом), после чего проигравшие покидают турнир, а победители опять делятся на пары, то таких раундов действительно было бы 10.

Задача №5

В наборе из нескольких монет одна фальшивая, либо больше, либо меньше, чем остальные. Известно, что за три взвешивания на чашечных весах без гирь ее можно отыскать. Каково максимальное число монет в этом наборе?

Замечание. Взвешивание на чашечных весах двух кучек позволяет определить, какая из них весит больше, или установить равенство их веса.

А. 3

Б. 9

В. 15

Г. 21

Д. 27

Решение

За k взвешиваний мы можем понять, какая монета фальшивая при наборе в (3k+1)/2 монет. То есть за 3 взвешивания, мы можем выяснить фальшивую монету в наборе из не более 14 монет. Поясним, откуда берется эта оценка сверху. Когда мы делаем взвешивание, весы попадают в одно из трех состояний (больше, меньше, равно). То есть после k взвешиваний, у нас может быть 3k различных исходов. Для n монет существует 2n исходов того, какая монета окажется фальшивой, и будет она легче или тяжелее настоящей. Таким образом, получается, что за k взвешиваний мы сможем отличить только 3k различных исходов, но на самом деле нам не обязательно различать исходы, легче или тяжелее фальшивая монета, если она уже найдена, поэтому за k взвешиваний мы можем понять, какая монета фальшивая при наборе в (3k+1)/2 монет. Важно, что про все эти монеты мы изначально ничего не знаем.

Посмотрим, сможем ли мы за 3 взвешивания определить фальшивую монету из 14 монет. Допустим первым взвешиванием мы взвесим на каждой чаше весов по 4 (или менее монеты), тогда в случае равенства весов, подозрительная монета будет находиться среди оставшихся монет. Их будет не менее 6. Тогда у нас останется 2 взвешивания, а как мы доказали, за 2 взвешивания, мы можем найти фальшивую монету в наборе из не более (32+1)/2 = 5 монет.

Так же первым взвешиванием мы могли положить на каждую чашу весов 5 или более монет. Тогда в случае неравенства, получаем, что фальшивой является одна из этих 10 (или более) монет, но про каждую подозрительную монету мы знаем, легче она или тяжелее настоящей, если является фальшивой, в зависимости от того, на какой чаше весов она лежит. За оставшиеся 2 взвешивания мы можем получить только 9 различных исходов (32), поэтому определить ее не сможем.

Получается, что с 14 монетами нам за 3 взвешивания не разобраться. Сейчас покажем, как мы сможем найти фальшивую монету из 13.

Первым взвешиванием возьмем по 4 монеты. Если получилось равенство, то будем считать, что у нас есть 8 настоящий монет и 5 подозрительных, про которые мы не знаем, легче или тяжелее они, чем настоящая. Вторым взвешиванием возьмем 3 подозрительные монеты с 3 настоящими. Если опять получилось равенство, то фальшивая среди 2х оставшихся, тогда взвесим одну из них с одной настоящей и определим она ли фальшивая или та, которую не взвешивали ни разу.

Если 3 подозрительные легче (для тяжелее все аналогично) 3х настоящий, то мы знаем, что фальшивая среди 3х подозрительных и она легче. Тогда возьмем и взвесим пару подозрительных монет друг с другом. Если одна из них легче, то она фальшивая, если они равны, то фальшивая третья.

Если в первом взвешивании получилось неравенство, то у нас есть 4 подозрительных легких монеты, 4 подозрительных тяжелых монеты и 5 настоящих монет. Тогда вторым взвешиванием взвесим на одной чаше 5 настоящих монет, а на другой 3 подозрительных легких и 2 подозрительных тяжелых. Если опять получилось равенство, тогда фальшивая монета среди 3х оставшихся монет – одной подозрительной легкой и 2х подозрительных тяжелых. Взвесим друг с другом 2 подозрительные тяжелые монеты. Если одна из них перевесила, то она фальшивая, если равенство – то фальшивая – оставшаяся легкая.

Если во втором взвешивании перевесили настоящие монеты, то фальшивая одна из 3х подозрительных легких со второй чаши весов, тогда возьмем пару из них и взвесим друг с другом, если одна из них легче, то она фальшивая, если равенство, то оставшаяся третья.

Если во втором взвешивании настоящие монеты легче, то фальшивая одна из 2х подозрительных тяжелых со второй чаши весов, тогда возьмем их и взвесим друг с другом, фальшивая та, которая перевесит.

Мы разобрали все возможные исходы и везде определили фальшивую монету.

Ответ: 13.

Комментарий

В этой задаче не было верного ответа среди предложенных, но так как нужно было найти максимальное возможное число монет в наборе, то следовало выбирать вариант Б (среди наборов из большего числа монет 15, 21, 27 найти монету за 3 взвешивания было никак нельзя). Тем не менее, в этой задаче около половины участников выбрали эти варианты. Вариант А так же встречался в некоторых работах, видимо, эти школьники не смогли придумать, как можно найти фальшивку в наборе из 9 монет.


Конкурс для учителей

1

2

3

4

5

Г

Б

Б

Г

Г

Комментарий

Так как в задачах из этой части нужно было только указать ответ без пояснений, мы не можем отследить, как был получен тот или иной неверный ответ. Разберем те задачи, в которых доля неверных ответов была значительной.

Задача №1

В примере все цифры заменены буквами, одинаковые – одинаковыми, разные – разными.

РОМА + РОМА + РОМА + РОМА = МАРТ

Найдите букву Т.

А. 0

Б. 2

В. 4

Г. 6

Д. 8

Решение

4*РОМА=МАРТ. Значит 1000<РОМА<2500 => 04000 => М>3.Если 4А<10, то остаток от деления на 10 числа 4М равен Р. Р=1 или 2. Подставляя все цифры от 4 до 9 вместо М видно, что подходит только М=8. Если 9<4А<20, то по аналогичным рассуждениям получаем, что М=5. Если 19<4А<37, то получаем М=5 и М=7. Значит М=5,7,8.Если М=8, то 4А<10 (А=0,1,2) , а значит Р=2 => так как РОМА<2500, то О<5, и остаток от деления числа 4*О+3 на 10 равен А. Подставляя вместо О цифры от 0 до 4,видно, что подходит только число 2 => О=2, но тогда 4*О+3>10, а значит М=9, противоречие. Если М=7, то 4А>29 (А=8,9) и Р=1, тогда 4*О+3>29 и 4*О+3 дает остаток 7 при делении на 10, что невозможно, противоречие. Значит М=5. Отсюда Р=1, 9<4*О+2<20, значит О=2,3,4. 9<4А<20 отсюда А=3,4. 4*О+2 четно, значит не может давать нечетный остаток при делении на 10, значит А=4, значит 4*О+2=14, отсюда О=3. 1354*4=5416. Значит Т=6.

Ответ: 6.

Задача №2

Программист печатает все натуральные числа, начиная с единицы, по одному в каждой строке. Однако на клавиатуре его компьютера не работает клавиша с цифрой «5». Поэтому он пропускает все числа, в которых встречается эта цифра. Какое число будет напечатано на 2014 строке.

А. 2787

Б. 2788

В. 2789

Г. 2677

Д. 2777

Решение

Заметим, что программист пишет числа в девятеричной системе исчисления. Переведем 2014 в 9-теричную систему счисления. 2014 в 9-теиричной равно 2677. Установим соответствие между цифрами 9-теричной и десятичной систем счисления.

0

1

2

3

4

5

6

7

8

9

0

1

2

3

4

---

5

6

7

8

В итоге 2 перейдет в 2, 6 в 7, а 7 в 8. Ответ: 2788.

Ответ: 2788.

Комментарий

Здесь среди неправильных чаще всего встречался вариант В.

Задача №3

Сколькими способами можно расставить 5 белых ладей, не бьющих друг друга, на доске 5×5?

А. 130

Б. 120

В. 24

Г. 720

Д. 25

Решение

Ладьи не бьют друг друга, если в каждом столбце и каждой строке стоит ровно одна ладья. В первую строку для ладьи есть 5 позиций. Во второй на одну меньше. И т. д. В последней строке – ровно одна позиция. Ответ: 5*4*3*2*1 = 120

Ответ: 120.

Комментарий

Среди неправильных в основном встречались варианты В и Д. Заметим, что в решении предполагалось, что доска зафиксирована на столе, а значит все расстановки, которые можно получить путем переворачивания доски считались, как различные.

Задача №4

1000 команд играют на «вылет» в турнире по волейболу. Какое минимальное количество матчей необходимо для нахождения победителя?

А. 10

Б. 499

В. 500

Г. 999

Д. 1000

Решение

За один матч отсеивается ровно одна команда. В итоге осталась только одна команда. Следовательно, было отсеяно ровно 999 команд. Значит, потребуется 999 матчей.

Ответ: 999.

Комментарий

Среди неправильных чаще всего встречался вариант А. По всей видимости, возникла путаница в том, что такое турнир на вылет. В турнире на вылет в каждом матче играют две команды, после чего победитель остается в турнире, а проигравший выбывает. Заметим, что если бы в турнире были раунды, в которых одновременно играют все команды друг с другом (например в первом раунде играют 500 пар друг с другом), после чего проигравшие покидают турнир, а победители опять делятся на пары, то таких раундов действительно было бы 10.

Задача №5

Среди 2014 монет, одинаковых по внешнему виду, одна фальшивая. Нам известно, что она отличается по весу от настоящих, но неизвестно, весит ли она меньше или же больше, чем остальные. Найдите минимальное количество взвешиваний, за которое можно определить фальшивую монету на чашечных весах без гирь.

Замечание. Взвешивание на чашечных весах двух кучек позволяет определить, какая из них весит больше, или установить равенство их веса.

А. 1007

Б. 10

В. 11

Г. 8

Д. 503

Решение

Когда мы делаем взвешивание, весы попадают в одно из трех состояний (больше, меньше, равно). То есть после k взвешиваний, у нас может быть 3k различных исходов. Для n монет существует 2n исходов того, какая монета окажется фальшивой, и будет она легче или тяжелее настоящей. Таким образом, получается, что за k взвешиваний мы сможем отличить только 3k различных исходов, но на самом деле нам не обязательно различать исходы, легче или тяжелее фальшивая монета, если она уже найдена, поэтому за k взвешиваний мы можем понять, какая монета фальшивая при наборе в (3k+1)/2 монет. (37+1)/2=1094<2014, следовательно, за 7 взвешиваний мы не можем определить какая монета фальшивая.

Покажем как за 8 взвешиваний определить фальшивую монета.

Предположим, что изначально мы знаем легче или тяжелее фальшивая монета, чем остальные. Понятно, что при любом взвешивании, мы будем на чаши весов класть по одинаковому количеству монет, иначе мы не получим никакой полезной информации после взвешивания. Любое взвешивание разбивает множество монет на 3 множества (монеты на левой и на правой чашах и те которые не участвовали при взвешивании). Множество монет, которые не участвовали во взвешивании всегда не меньше множества монет которые лежат на одной чаше весов. И после любого взвешивания однозначно определяется множество, содержащее фальшивую монету. То есть, если изначально у нас было N монет, то после взвешивания остается не более  подозрительных монет ( – целая верхняя часть от числа ). Так как мы не знаем легче или тяжелее фальшивая монета, то в самом плохом случае после одного взвешивания подозрительных монет останется больше чем  (ситуация при которой весы не будут в равновесии. Без ограничения общности положим, что левая чаша перевесила правую). Тогда при следующем взвешивании мы меняем местами множество монет на левой чаше с множеством монет не участвовавших во взвешивании, (если в множестве монет не участвовавших во взвешивании монет больше на 1ну чем на левой чаше, то берем из этого множества на одну монету меньше, а ту монетку которую мы не взяли добавим в множество монет которое лежало на левой чаше) и проводим второе взвешивание. Если весы оказались в равновесии, то фальшивая монета тяжелее настоящей, и не участвовала во втором взвешивании, если левая чаша перевесила, то фальшивая монета на правой чаше, и она легче настоящей. Ситуация при которой правая чаша перевесила левую невозможна, так как тогда фальшивая монета в правой чаше, и из второго взвешивания она тяжелее настоящей, а из первого взвешивания легче настоящей. В итоге за 2 взвешивания в самом плохом случае остается не более  подозрительных монет. Далее взвешивания будут проводиться по алгоритму, описанному в самом начале. Таким образом, необходимо и достаточно 8 взвешиваний.

Ответ: 8.

Комментарий

Эта задача оказалась самой сложной в первой части. Самым популярным был вариант Б. Видимо, только за такое количество взвешиваний многие смогли найти монету.


 



Предварительный просмотр:

Разбор задач шестой части заданий


Разбор задач шестой части заданий

7 класс

Задача №1

Можно ли с помощью измерений обычной линейкой обнаружить, что из стопки бумаги высотой 5 см, содержащей 500 листов, вынули: 1) ровно 1 лист; б) ровно 10 листов; 3) четверть стопки (с точностью до 10 листов)?

Решение

Из условия вытекает, что толщина одного листа равна 5 см:500 = 0,01 см, то есть 0,1 мм, что нельзя измерить с помощью обычной линейки. Поэтому измерение любого количества листов с точностью до 1 листа невозможно.

Четверть стопки, то есть 125 листов имеет толщину 12,5 мм, а 10 листов — 0,1 мм10 = 1 мм. Поэтому уменьшение толщины стопки на 12,5 мм с точностью до 1 мм с помощью измерений обычной линейкой обнаружить можно.

Ответ: 1) Нет; 2) нет; 3) да.

Комментарий

В этой задаче практически все верно посчитали толщину листа, но верные выводы сделало меньшинство. С пунктом 1) справились почти все. Школьники отлично знают, что деление на обычной линейке равно 1 мм, а значит с ее помощью пропажу 1 листа толщиной 0,1 мм никак не обнаружить. А вот в пункте 2) у многих возникли сложности. Почти все считали, что 10 листов имеют толщину 1 мм, поэтому на первый взгляд кажется, что их пропажу можно обнаружить, но мало кто догадался, что пропажу 10 листов мы  никак не сможем отличить от, например, пропажи, 11 листов, а у нас спрашивается, можем ли мы обнаружить, что вынули ровно 10 листов, а не сам факт, что из стопки что-то убрали. Хотя указанная в 3) пункте точность измерений могла натолкнуть на мысль, что обычной линейкой мы не сможем определить пропажу точного числа листов. С пунктом 3) опять справились почти все.

Отдельно стоит отметить работы, в которых были указаны только ответы можно или нет без объяснений. Этого было не достаточно. Во всех подобных задачах следует обосновывать, почему что-то сделать нельзя или, если можно, приводить способ.

Задача №2

Квадратный лист бумаги сложили по линии, параллельной одной из границ листа, на две равные части, а потом ещё раз по линии, параллельной одной из границ образовавшихся частей, на две равные части. Затем образовавшийся кусок бумаги разрезали по прямой. Сколько частей могло образоваться при этом?

Решение

При указанном в задаче сложении квадратного листа бумаги может получиться кусок прямоугольной формы, состоящий из четырёх слоёв или кусок квадратной формы.

На рисунке 1 изображён квадратный лист бумаги, на котором вертикальные тонкие линии изображают линии сгиба, а штриховые линии — линии разреза бумаги. На рисунке 2 изображён квадратный лист бумаги, на котором штриховыми линиями отмечены линии, по которым разрезается бумага.

В обоих случаях могут получиться 2, или 3, или 4 части, или 5 частей в зависимости от направления разреза.

Так как в первом случае линии разреза в каждой четверти листа между двумя соседними линиями сгиба попарно симметричны, то не могло образоваться более 5 частей.

Невозможность получения большего количества частей во втором случае следует из того, что след от разрезов симметричен относительно осей симметрии квадрата и его центра.

Ответ: От двух до пяти частей.

Комментарий

Ситуация с этой задачей аналогична 5-му и 6-му классам. (В 5-м была эта же задача, но указывалась, что 2 раза складываем по параллельным сгибам, а в 6-м, что получается квадрат). Здесь же нужно было рассматривать оба случая, но на типичные ошибки это не повлияло.

Верных решений в этой задаче было очень мало. Большинство находили не все случаи. Пропускали самые разные: кто-то указывал один вариант и на этом останавливался, кто-то 2, кто-то 3. Чаще всего находили разрезания на 2 и 5 частей. В некоторых работах школьники указывали только ответ, но не приводили способ, как нужно резать, чтобы получить указанное количество частей. И почти никто из получивших все возможные варианты, не смог объяснить, почему других не бывает.

В таких задачах, важно научиться грамотно организовать перебор вариантов. На первый взгляд кажется, что вариантов разрезать лист бесконечное количество. Важно понять, какие варианты отличаются друг от друга, а какие на самом деле одинаковые, т.е. никак не влияют на количество получившихся частей.

Задача №3

По окружности расположено n кружочков, занумерованных числами 1, 2, …, n. Будем закрашивать кружочки, начиная с кружочка с номером 2, через один незакрашенный кружочек (кружочки с номерами 2, 4, 6, …) до тех пор, пока останется один незакрашенный. Каков его номер при n, равном: 1) 261; 2) 520?

Решение

Если кружочков было бы 256 = 28 или 512 = 29, то незакрашенным остался бы кружочек с номером 1. При каждом прохождении окружности количество незакрашенных кружочков уменьшается вдвое и оставшиеся кружочки разбиваются на пары. Так как в паре закрашивается второй кружочек, то кружочек с номером 1 никогда не будет закрашен.

Если кружочков 261 = 256 + 5, то, закрасив 5 кружочков с номерами 2, 4, 6, 8, 10, получим 256 незакрашенных кружочков, и первым из них будет кружочек с номером 11. Он и останется после всех закрашиваний.

Если кружочков 520 = 512 + 8 = 29 + 8, то, закрасив 8 кружочков с номерами 2, 4, 6, 8, 10, 12, 14, 16, получим 512 незакрашенных кружочков, и первым среди них будет кружочек с номером 17. Он и будет последний незакрашенный.

Ответ: 1) 11; 2) 17.

Комментарий

Эта задача оказалась самой трудной в олимпиаде, верных решений было крайне мало. Чаще всего возникала путаница с пониманием, как именно закрашиваются кружочки, почему то большинство школьников не смогли внимательно прочитать условие и закрашивали через один кружочек, а не через один незакрашенный. Соответственно в первом пункте они получали ответ 261. Во втором же пункте был разброс вариантов, кто-то писал, что останутся незакрашенными все нечетные, кто-то, что ответ 519 (видимо как самый большой номер среди нечетных). Мало кто из школьников приводил внятные объяснения, откуда берутся эти ответы.

Некоторые пытались найти закономерность при меньшем числе кружков. Сам этот подход имеет место на существование, но нужно четко понимать, почему найденная закономерность будет выполняться для всех чисел. Здесь же встречались варианты, что если для 7 кружков последний незакрашенный 7, то и для любого нечетного числа так будет. (На самом деле, мы видим, что это не так). В одной из работ школьник нашел закономерность, что для нечетного числа кружков на каждом круге первый закрашиваемый кружок сдвигается к середине по порядку, а для каждого четного мы всегда пропускаем 1. Эти закономерности тоже не верны.

Задача №4

Виталий закупает газеты по 4 зеда за номер, которые затем реализует по 5 зедов (зед — условная денежная единица). Если у него остаются нераспроданные газеты, то он их сдаёт на следующий день в киоск по 2 зеда за номер.

  1. Какое количество проданных в первый день газет обеспечивает: а) неубыточность деятельности Виталия; б) прибыльность его деятельности?
  2. На сколько процентов нужно увеличить цену реализации газет, чтобы продажа половины закупленных по цене 4 зеда газет обеспечивала прибыльность деятельности Виталия?

Решение

  1. Пусть Виталий закупает ежедневно х газет, а продаёт в тот же день у (у  х) газет. Тогда на следующий день он сдаёт в киоск х – у газет. Его расходы составляют 4х (зедов), а получает он 5у + 2(х – у) = 3у + 2х (зедов).
  1. Деятельность Виталия будет неубыточной, если доходы не меньше расходов, то есть если 3у + 2х  4х, или у  х. Следовательно, Виталий должен продавать по цене 5 зедов не менее  купленных газет.
  2. Деятельность Виталия будет прибыльной, если его доходы превышают расходы, то есть если у > х. Следовательно, Виталий должен продавать по цене 5 зедов более  купленных газет.
  1. Пусть по-прежнему Виталий закупает ежедневно х газет, а продаёт их по цене а зедов за номер. Его расходы составляют 4х (зедов), а получает он при продаже половины закупленных газет а + 2 (зедов). Прибыльной будет деятельность Виталия, если 4х < а + 2, или а > 6. Следовательно, цену реализации нужно увеличить более чем на 6 – 5 = 1 (зеда) или более чем на  = 20%.

Ответ: 1) а) Не менее  закупаемых газет; б) более  закупаемых газет.
2) Более 20%.

Комментарий

В этой задаче тоже было очень много неверных решений. Во первых, даже при верно составленных и решенных уравнениях, школьники путаются в формулировках «не более», «не менее», «равно». В первом пункте многие выписывают ответ 2/3 от проданных газет, а не «не менее 2/3».

Во вторых, как и во многих предыдущих задачах, школьники часто указывают ответ без каких-либо объяснений, а этого не достаточно.

В третьих, школьники забывают про то, что непроданные газеты можно сдать на следующий день, отсюда берутся ответы «более 4/5» в пункте Б и «более 40%» в пункте В.

Кроме этого, некоторые школьники решают задачу для конкретного числа газет, что тоже приводит к разным неверным ответам.


8 класс

Задача №1

Торт, имеющий форму прямоугольного параллелепипеда с квадратным основанием, сверху и с боков равномерно покрыт шоколадной глазурью. Как разрезать этот торт на 7 частей с одинаковой массой глазури на каждом?

Решение

Если разделить границу квадрата на 7 равных по длине частей и соединить эти точки с центром квадрата О, то квадрат разобьётся на треугольники и, быть может, на четырёхугольники, площади которых равны. Площадь каждого треугольника равна , где а — сторона квадрата.

Если разбить четырёхугольник на два треугольника диагональю, соединяющей центр квадрата с вершиной квадрата, то можно показать, что площадь четырёхугольника также равна .

Площади участков боковой поверхности торта, соответствующие частям, образованным по краям верхнего основания, также равны. Доказательство аналогично предыдущему.

Учитывая, что толщина шоколадной глазури мала, то, в соответствии с точностью измерений длин на поверхности торта, можно считать, что массы глазури на боковой поверхности торта тоже равны.

Приведенный способ деления торта обеспечивает и равенство масс кусков, то есть полную справедливость деления.

Комментарий

Больше половины участников не приводили никакого решения этой задачи. Из тех, кто приводил, целиком верных решений было крайне мало. Во многих работах указывался верный способ разрезания без доказательства, что площади глазури на всех кусках при указанном разрезании равны. Почему-то школьники часто решали, что торт имеет форму куба, в данном случае, это никак не влияло на доказательство, но такое решение нельзя считать полностью верным.

Кроме этого часто в работах был нарисован способ разрезания, но никаких пояснений к нему не было (по картинке похоже, что разрезы от центра, и что граница разбита на равные по длине части), но без пояснений такие картинки оценивались в небольшое количество баллов.

Задача №2

Участок квадратной формы разделили на четыре участка прямоугольной формы, имеющих общую угловую точку. Площади трёх из них относятся как 1:2:3. Найдите площадь четвёртого участка, если длина забора вокруг всего участка равна 240 м и известно, что четвёртый участок имеет наименьшую площадь из всех 4-х частей.

Решение

Обозначим через х и у (у  х) размеры участка наибольшей площади. Данный участок и его разбиение на 4 прямоугольные части, соответствующие условию, изображены на рисунке. Так как периметр участка равен 240 м, то сторона квадрата на рисунке равна 60 м.

Наименьшую площадь S имеет прямоугольник, не имеющий общей стороны с прямоугольником, имеющим наибольшую площадь.

Из условия и введенных обозначений следуют следующие равенства:

S1 = 2S2, S1 = 3S3, S1 = xy, S2 = y(60 – x), S3 = x(60 – y).

Следовательно, имеем систему уравнений:

 или  или  или

Искомая площадь равна S = (60 – 40)(60 – 45) = 300 (м2).

Случай второй:

S1 = 2S2, S1 = 3S, S1 = xy, S2 = y(60 – x), S = (60 – x)(60 – y).

Аналогично составляем систему уравнений. Решая систему уравнений, получаем S = (60 – 40)(60 – 36) = 480 (м2).

Случай третий:

S2 = 2S3, S2 = 3S, S2 = y(60 – x), S3 = x(60 – y), S = (60 – x)(60 – y).

Аналогично составляем систему уравнений. Решая систему уравнений, получаем S = (60 – 45)(60 – 36) = 360 (м2).

Ответ: 300 м2, 360 (м2), 480 (м2).

Комментарий

В этой задаче практически не было верных решений. Обычно школьники составляли одну возможную систему уравнений и решали ее, соответственно получалась либо площадь 300 м2, либо 360 м2, реже 480 м2.

Кроме этого, очень многие почему-то не верно читали условие и считали, что все площади относятся как 1:2:3:4, а отсюда получали ответ, что площадь самой меньшей части 360, тем самым получая один из возможных верных ответов.

Так же в этой задаче часто встречались ошибки в вычислениях.

Как всегда, во многих работах указывался только ответ без объяснения, откуда он был получен. В очередной раз отметим, что только ответа было не достаточно.

Задача №3

В классе часть учеников изучает английский язык, часть — французский. Изучающих английский язык в два раза больше изучающих французский. Девочек, изучающих английский язык, столько же, сколько всего мальчиков в классе, а вместе девочек, изучающих английский язык, и мальчиков, изучающих французский язык, в два раза больше, чем всех остальных. Сколько в классе мальчиков, если количество учеников в классе не меньше 20, но не больше 30?

Решение

Обозначим через ам и ад количества соответственно мальчиков и девочек в классе, изучающих английский язык, а через fм и fд — количества соответственно мальчиков и девочек в классе, изучающих французский язык.

Из условия задачи следуют равенства:

ам + ад = 2(fм + fд), ад = ам + fм, fм + ад = 2(aм + fд).

Если в третье равенство подставить выражение для ад из второго и преобразовать полученное равенство, то будем иметь:

ам + 2fм = 2fд + 2ам или 2fм = 2fд + ам.

Если полученное равенство сложить почленно с первым равенством и преобразовать его, то придём к следующему равенству:

ад = 4fд.

Следовательно, количество девочек, равное ад + fд = 5fд, делится на 5.

Рассматривая все возможные значения количества девочек в классе, найдём, пользуясь приведенными равенствами, соответствующие количества учащихся в классе.

ад + fд

ад 

fд

ам + fм

Количество учеников

5

4

1

4

9

10

8

2

8

18

15

12

3

12

27

20

16

4

16

36

Так как количество учеников в классе не меньше 20, но не больше 30, то количество девочек в классе может равняться только 15. Тогда значения ад = 12, fд = 3, ам = 6, fм = 6 удовлетворяют всем условиям задачи. В классе 12 мальчиков.

Ответ: 12 мальчиков.

Комментарий

Эта задача оказалась самой легкой во второй части, многие с ней справились. Но нашлись те, кто отвечал хоть и верно, но на другой вопрос: «Сколько всего учеников в классе?» В некоторых работах школьники правильно определяли общее число учеников в классе, но делали не правильный вывод, сколько из них мальчиков. (Часто получалось 9).

Встречались и другие не верные ответы, они все получались из неких предположений, которых не было в условии задачи (например, что общее число школьников в классе 25), при этом выполнимость всех условий не проверялась.

Так же часто встречались только ответы без объяснений, либо приводилось количество мальчиков и девочек, изучающие каждый язык, но без доказательства, почему не бывает другой разбивки, удовлетворяющей условию.

Задача №4

По окружности расположено n кружочков, занумерованных числами 1, 2, …, n. Будем закрашивать кружочки, начиная с кружочка с номером 2, через один незакрашенный кружочек (кружочки с номерами 2, 4, 6, …) до тех пор, пока останется один незакрашенный. Каков его номер?

Решение

Если n = 2k, то останется незакрашенным кружочек с номером 1. Это следует из того, что обход по окружности приводит к закрашиванию половины кружочков. При этом закрашенным будет кружочек, стоящий перед кружочком с номером 1. После первого обхода количество незакрашенных кружочков будет равняться 2k – 1 . Продолжая этот процесс, придём к паре незакрашенных кружочков, в которой первым будет кружочек с номером 1.

Если n = 2k + р, где 0 < p < 2k, то последним незакрашенным останется кружочек с номером 2р + 1. Чтобы доказать это, закрасим р кружочков (2, 4, 6, …, 2р) по правилу, указанному в условии. Вдоль окружности незакрашенными останется 2k кружочков. Первым после незакрашенных будет кружочек с номером 2р + 1. Он и останется незакрашенным после выполнения всех закрашиваний.

Ответ: 2р + 1, если n = 2k + р, где 0  p < 2k.

Комментарий

Эта задача оказалась самой сложной во всей олимпиаде. Верных решений практически не было. Чаще всего возникала путаница с пониманием, как именно закрашиваются кружочки, почему то большинство школьников не смогли внимательно прочитать условие и закрашивали через один кружочек, а не через один незакрашенный.

Большинство школьников писали, что при четном и нечетном n будут разные варианты. Чаще всего указывался вариант, что при четном n будет оставаться кружочек с номером 1, а при нечетном с номером n. Мало кто из школьников приводил объяснения, откуда берутся эти ответы.

Некоторые пытались найти закономерность на маленьком числе кружков, но мало кто смог эту  закономерность обнаружить.

Так же в некоторых работах школьники давали ответ только для конкретного числа кружочков. Например, для 4 или 19, не пытаясь найти общее решение.


9 класс

Задача №1

Катер проплыл 60 км по течению реки и 48 км против течения, затратив на весь путь 6 ч. Найдите отношение собственной скорости катера к скорости течения, если эти скорости выражаются целыми значениями км/ч.

Решение

Пусть х (км/ч) — собственная скорость катера, у (км/ч) — скорость течения. Тогда, по условию,  +  = 6 или  +  = 1. Так как в последнем равенстве оба слагаемые в левой части положительны, то приведенное уравнение имеет решение, если одно из этих слагаемых не меньше , а другое — не больше . Эти условия выполняются, если х + у  20, а 8 < х – у  16 или 10 < х + у  20, а х – у  16. В первом случае из свойств неравенств получаем: у  2, а во втором — у  2. Если у = 2, то уравнение принимает следующий вид:  +  = 1 или х2 = 18х. Условию задания удовлетворяет корень х = 18.

Значения у = а, где а > 2, приводят к уравнению  +  = 1 или к уравнению х2 – 18х – 1 +2а – а2 = 0, которое целых решений не имеет. Точно так же, если у = 1, то уравнение не имеет целых решений. Итак, искомое отношение равно 18:2 = 9:1.

Ответ: 9:1.

Комментарий

С этой задачей справилось около трети участников. Радует, что большинство верных ответов были подкреплены решениями.

Получение неправильных ответов чаще всего было связано с ошибками в вычислениях. Кто-то неверно сократил дробь, кто-то посчитал, что если сумма двух слагаемых равна 0, то и каждое из слагаемых должно быть равно 0 (видимо, школьник забыл про существование отрицательных чисел).

Стоит отметить несколько решений, в которых школьники перебирают различные целочисленные значения скоростей, но не проверяют, что эти скорости должны соответствовать всем условиям задачи, поэтому получают не только верный ответ, но и несколько побочных не верных. Кроме этого, в некоторых работах участники отвечают не на тот вопрос. Например, они находят скорость катера. Нужно внимательно читать условие и вопрос задачи.

Задача №2

Вокруг территории склада, имеющего форму выпуклого шестиугольника, вспахали заградительную полосу шириной 2 м. Какова площадь этой полосы, если длина ограды территории склада 450 м?

Решение

Изобразим территорию склада в виде шестиугольника. Тогда изображением заградительной полосы будет множество точек вне шестиугольника, удалённых от шестиугольника на расстояние не более 2 м (см. рис. 1).

Это множество состоит из прямоугольников, измерения которых равны 2 м, и длинам соответствующих сторон шестиугольника и секторов круга с радиусом 2 м, центральные углы которых дополняют соответствующие углы шестиугольника до 180° (см. рис. 2).

Обозначим углы шестиугольника через β1, β2, …, β6. Тогда 180° – β1 + 180° – β2 + … + 180° – β6 = 6180° – (β1 + β2 + … + β6.) = 6180° – (6 – 2)180° = 360°.

Следовательно, указанные секторы вместе составляют круг.

Искомая площадь S определяется формулой S = 2р + πr2, где р — периметр шестиугольника. Итак, S  913 м2.

Ответ:  913 м2.

Комментарий

Очень многие школьники, получившие верный ответ в этой задаче решали ее для склада, представляющего из себя правильный шестиугольник, хотя в условии было дано только то, что он выпуклый. В данном случае, на ответ это не повлияло, но в таких решениях стоило привести объяснения, почему площадь полосы неправильного шестиугольника будет такой же, как и для правильного.

Самым распространенным неправильным ответом был 900 (450*2). Школьники просто не учитывали те куски полосы, которые представляют из себя сектора круга. Были так же и такие работы, в которых участники не смогли понять, что из себя представляют эти куски на углах шестиугольника, отсюда получались разные неверные ответы.

Задача №3

Завод должен переслать заказчику 1100 деталей. Детали упаковываются в ящики трёх видов: по 70, 40 или 25 деталей в каждом. Стоимость пересылки одного ящика соответственно 500, 250 или 175 зедов (зед — условная денежная единица). Сколько ящиков и какого вида должен использовать завод, чтобы стоимость пересылки была наименьшей? Недогрузка ящиков не допускается.

Решение

Стоимости пересылки одной детали в ящиках соответственно составляют  зедов. Поскольку  то выгоднее всего пересылать детали в ящиках по 40 штук, менее выгодно — по 25 штук и ещё менее выгодно упаковывать детали в ящики по 70 штук. Но в ящики по 40 штук нельзя упаковать без недогрузки 1100 деталей, поэтому приходится искать наибольшее количество деталей, которые можно переслать в ящиках по 40 штук. Это количество следует искать среди чисел, близких к 1100 и кратных 40, то есть среди чисел 1080, 1040, 1000, … Из последовательности 1080, 1040, 1000, … первым числом, которое нам подходит, является число 1000, так как в этом случае остаётся 100 деталей, которые можно разместить в четырёх ящиках по 25 штук.

Итак, выгоднее всего взять 25 ящиков по 40 деталей в каждом и 4 ящика по 25 деталей.

Ответ: 25 ящиков по 40 деталей в каждом и 4 ящика по 25 деталей.

Комментарий

Эта задача оказалась самой легкой во втором блоке, тем не менее справились с ней только около половины участников. Как всегда, нашлись работы, в которых школьники указывали только верный ответ без каких-либо пояснений. Часто в качестве ответа указывалась сумма в 6950 зед (это минимальная стоимость пересылки). Очевидно, что получив минимальную стоимость, школьники знали, сколько и каких ящиков нужно использовать для пересылки, но все равно отвечали не на тот вопрос.

Было много работ, в которых перебирались различные комбинации ящиков, в которых можно было переслать детали, для каждой из них считалась стоимость пересылки, и выбирался самый дешевый вариант. Почти везде перебор был не полный, поэтому даже если получался верный ответ, такое решение нельзя засчитать за верное. Часто верного ответа вовсе не было среди рассматриваемых, поэтому и самую оптимальную пересылку найти не удавалось.

Так же в некоторых работах школьники игнорировали условие, что недогрузка ящиков не допускается, соответственно они тоже получали не верный ответ.

Задача №4

Какое наименьшее количество одинаковых монет нужно иметь, чтобы полностью покрыть ими квадрат, сторона которого вдвое больше диаметра монеты?

Решение

Выберем половину диаметра монеты за единицу измерения. Задача сводится к покрытию квадрата со стороной 4 см кругами, диаметры которых равны 2 см.

На рис. 1 изображено покрытие диагонали квадрата со стороной 4 см тремя кругами, радиусы которых 1 см, а центры О1, О2, О3 расположены так, что АО1 = 1, СО2 = 1, АО3 = СО3.

Так как длина диагонали квадрата АС = 4 < 6, то указанные круги покрывают диагональ.

Рассмотрим теперь отрезок MN, где М — точка пересечения одного из построенных кругов со стороной AD, N — точка пересечения другого со стороной DC (рис. 2).

По построению, АМ = CN = . Тогда MD = ND = 4 –  (см), а MN = (4 – ) < 4 (см). Следовательно, круги с центрами в точках О4 и О5 (4 = 1 см, 5 = 1 см) покрывают отрезок MN. Они также касаются диагонали АС в точках, являющихся концами радиусов кругов с центрами в точках О1 и О2. Следовательно, построенные круги покрывают трапецию АСNM.

Рассмотрим отрезок DK, где K — точка пересечения диагонали BD с отрезком MN (см. рис. 3). По построению, DK =  =  = 2 – 1 < 2 (см). Следовательно, круг с центром в точке О6 и с радиусом 6 = 1 (см) покрывает отрезок DK. Он вместе с кругами с центрами О4 и О5 покрывает треугольник MND. Это следует из того, что  <  (см), где Р — точка пересечения круга с центром в точке О4 со стороной CD:

DP = DC – CN – PN = 4 –  –  = 4 – 2 (см).

Построив круги, симметричные кругам с центрами О4, О5 и О6 относительно прямой АС, получим покрытие квадрата со стороной 4 см девятью кругами с диаметром 2 см.

Невозможность покрытия восемью кругами следует из приведенных соображений. Для покрытия треугольника MDN нужно не менее трёх кругов. Для покрытия диагонали требуется не менее трёх кругов. Следовательно, для покрытия квадрата требуется не менее 9 кругов.

Ответ: Девять.

Комментарий

С этой задачей не справился почти никто. Некоторые школьники догадались, что кругов нужно 9 и даже привели верный пример покрытия, но вот доказательство, почему квадрат невозможно покрыть меньшим количеством вызвало затруднения.

Очень многие приводили примеры покрытия на 10, 12 или 13 кружочков, до меньшего количества они не додумались.

Кроме этого, во многих работах были приведены оценки снизу с доказательством для 4, 5 или 6 кружочков. Доказательства были верными, но их было не достаточно. Некоторые даже приводили в работах покрытия для меньшего 9 числа кружочков, но все эти покрытия были не полными.




Предварительный просмотр:

Разбор задач четвертой части заданий


Разбор задач четвертой части заданий

7 класс

6

7

8

9

10

А

Б

Б

А

В

Задача №6

Сравните длину пути l, преодолеваемого концом минутной стрелкой механических часов за 10 минут, с длиной этой стрелки l1.

А. l > l1.

Б. l = l1.

В. l < l1.

Г. Сравнить нельзя.

Решение

За 10 минут минутная стрелка сделала  оборота. Следовательно, её конец пройдет путь, равный = , что больше , так как .

Ответ: A. .

Комментарий

В этой задаче очень много неверных ответов, хотя единственное, что нужно было вспомнить – это формулу длины окружности. Среди неверных ответов часто встречался ответ Б, возможно возникла путаница длины пути и перемещения конца минутной стрелки (перемещение действительно равно длине стрелки).

Задача №7

На диаграмме изображена начисленная фирмой общая сумма заработной платы всем своим сотрудникам в январе, феврале и марте 2013 года (зед — условная денежная единица). В январе на фирме работали 15 сотрудников, в феврале — 18, а в марте — 25. Как изменилась средняя начисленная заработная плата в этой фирме в марте по сравнению с январём?

А. Увеличилась более чем на 4000 зедов.

Б. Увеличилась менее чем на 4000 зедов.

В. Уменьшилась более чем на 4000 зедов.

Г. Уменьшилась менее чем на 4000 зедов.

Решение

Из диаграммы видно, что в январе общая заработная плата на фирме составляла 120 тыс. зедов, а в марте — 280 тыс. зедов. Так как в январе на фирме работали 15 сотрудников, а в марте — 25, то средняя зарплата одного сотрудника в январе составила 120 000:15 = 8 000 (зедов), а в марте — 280 000: 25 = 11 200 (зедов), то есть она увеличилась на 3 200 зедов, что меньше 4 000 зедов.

Ответ: Б. Увеличилась менее чем на 4000 зедов.

Комментарий

Большинство школьников с задачей справились. В одной из работ, где школьник приводил решения в том числе на тестовые задачи (этого делать не требовалось, но такие работы помогают понять типичные ошибки), ответ А был получен за счет того, что он посчитал, что на графике средняя зарплата одного работника в зедах, а сравнить нужно общий зарплатный фонд. Видимо все остальные неверные ответы были получены так же из-за не внимательно прочитанных условий.

Задача №8

В классе есть девочки и мальчики. Какое из утверждений может быть истинным?

А. Каждый мальчик ниже какой-то девочки, и каждая девочка не выше любого мальчика.

Б. Каждый мальчик ниже какой-то девочки, и какая-то девочка не выше любого мальчика.

В. Каждый мальчик не выше какой-то девочки, и любая девочка ниже любого мальчика.

Г. Какой-то мальчик ниже любой девочки, и какая-то девочка не выше любого мальчика.

Решение

Каждое из утверждений, приведённых в ответе, состоит из двух более простых утверждений. Истинным может быть то утверждение, в котором оба простых утверждения не противоречат друг другу.

Утверждение А не может быть истинным, так как если каждый мальчик ниже какой-то девочки, то каждая девочка не может быть ниже любого мальчика или совпадать по росту с любым мальчиком: одна девочка выше всех мальчиков.

Утверждение В не может быть истинным, так как если каждый мальчик не выше какой-то девочки, и любая девочка не может быть ниже любого мальчика.

Утверждение Г не может быть истинным, так как если какой-то мальчик ниже любой девочки, то все девочки не могут быть не выше любого мальчика.

Утверждение Б истинно, так как если каждый мальчик ниже какой-то девочки, то это не исключает того, что какая-то другая девочка не выше любого мальчика.

Ответ: Б.

Комментарий

Больше половины участников решили задачу верно. Среди неправильных чаще всего встречался ответ Г. Он не может быть верным, потому что если мы возьмем того самого мальчика, который ниже любой девочки, то какую бы девочку мы не взяли, она будет выше его.

Задача №9

Робот начинает движение в некоторой точке, в начале движения он выбирает направление перемещения. Далее робот движется прямолинейно 10 м, затем поворачивает на 90° вправо или влево и движется прямолинейно 10 м, далее снова поворачивает на 90° вправо или влево и движется прямолинейно 10 м и т. д. Сколько различных расстояний может отделять робота от начала пути, если он остановился на месте 6-го поворота?

А. 3.

Б. 4.

В. 5.

Г. 6.

Решение

Изобразим первое перемещение робота горизонтальным отрезком. Тогда дальнейшее движение робота будет проходить по сторонам квадратной сетки, изображённой на рисунке, со стороной 10 м. Точка О — начало движения.

Возможные варианты расположения робота непосредственно перед 6-м поворотом отмечены на рисунке жирными точками. Возможные расстояния от начала движения до места 6-го поворота могут равняться: ОА, ОВ, ОС. Всего их 3.

Ответ: А. 3.

Комментарий

В этой задаче опять большой разброс разных ответов, все ответы встречались очень часто. Чаще всего встречался вариант Г. Скорее всего школьники не учитывали, что разные маршруты в итоге могут удалить робота от начала на одинаковые расстояния. Ошибиться было достаточно просто, если пытаться решить задачу в уме, но можно было легко перебрать все возможные маршруты робота. Если он остановился на месте 6-го поворота, значит он поворачивал ровно 5 раз, с каждым поворотом количество маршрутов увеличивалось в 2 раза, то есть всего было 32 маршрутов (2 * 2 * 2 * 2 * 2 = 32), некоторые из них приводили робота в одну и ту же точку.

Задача №10

Имеется набор фигур, составленных из спичек. На рисунке показано, как следующая фигура составляется из предыдущей. Длина каждой стороны треугольника равна длине спички. Каков номер фигуры, для составления которой потребовалось 545 спичек?

А. №170.

Б. №169.

В. №136.

Г. №138.

Решение

Каждая следующая фигура получается из предыдущей добавлением фигуры №1, причём одна сторона предыдущей фигуры совпадает с одной стороной добавляемой фигуры № 1. Так как фигура №1 содержит 5 спичек, то каждая последующая фигура будет содержать на 4 спички больше, чем предыдущая. Количества спичек, необходимых для составления фигур, соответственно равны: 5, 9, 13, 17, … . Эти числа при делении на 4 дают в частном номер фигуры, а в остатке 1: 5 = 41 + 1, 9 = 42 + 1, 13 = 43 + 1, 17 = 44 + 1, … . Следовательно, чтобы найти искомый номер, нужно от заданного количества спичек — числа 545 — отнять 1 и полученную разность разделить на 4: (545 – 1):4 = 136.

Ответ: В. №136.

Комментарий

С задачей справились практически все участники, неправильных ответов почти не было.


8 класс

6

7

8

9

10

В

Б

Б

А

Б

Задача №6

Из конечных пунктов маршрута выехали одновременно навстречу друг другу автобус и маршрутное такси. Автобус преодолевает маршрут за 1 ч 40 мин, а маршрутное такси — за 1 ч 10 мин. Через сколько примерно минут после выезда автобус и маршрутное такси встретятся, если скорости их движения мало изменяются в пути? Выберите наиболее точное значение.

А. Через 75 мин.

Б. Через 55 мин.

В. Через 40 мин.

Г. Через 30 мин.

Решение

Обозначим расстояние между конечными пунктами маршрута через s (км). Тогда средняя скорость автобуса равна  (км/ч), средняя скорость такси —  (км/ч), а средняя скорость их сближения —  +  =  (км/ч). Если бы их движение было равномерным, то их встреча произошла бы через s: =  (ч) или через 60  41 (мин). Но можно считать, что за промежуток времени длиной 10 мин каждый из них проходит примерно одно и то же расстояние. Поэтому из приведенных ответов наиболее подходящим является ответ В.

Ответ: В. Через 40 мин.

Комментарий

С задачей справилось большинство участников. Среди неверных ответов лидировал вариант Г.  Нашлись и те, кто выбрал вариант А, хотя совершенно очевидно, что встретятся они намного раньше, чем доедут до конца маршрута, а 75 минут больше, чем время, необходимое маршрутному такси для преодоления всего расстояния.

Задача №7

Какое наименьшее количество автобусных маршрутов, соединяющих два города, каждый из которых позволяет добраться из одного города в другой и обратно, необходимо для того, чтобы из каждого из n городов можно было добраться автобусом в любой другой, сделав не более одной пересадки?

А. n.

Б. n – 1.

В. n – 2.

Г. .

Решение

Можно один из городов, например N, соединить автобусными маршрутами с остальными (n – 1) городами. Для этого понадобится n – 1 автобусный маршрут. Тогда из любого из n городов можно попасть в любой другой, сделав пересадку в городе N.

Это количество маршрутов минимально. Действительно, пусть n городов соединены n – 2 маршрутами так, что из любого города можно добраться автобусом в любой другой, сделав не более одной пересадки (даже если маршрутов было меньше, всегда можно добавить маршруты так, чтобы их было n – 2). Отбросим все города, из которых выходит только один маршрут, вместе с маршрутами, выходящими из них. Пусть таких городов k  0. Тогда осталось n – k = m городов. Так как два города не могут быть соединены маршрутами только между собой, то количества отбрасываемых городов и маршрутов одинаковы, то есть маршрутов осталось n – 2 – k = m – 2.

Из условия следует, что осталось не менее двух городов. В противном случае все отброшенные города были связаны автобусными маршрутами с оставшимися (не остаться не могло). И количество маршрутов было n – 1, а не n – 2.

Докажем методом от противного, что из любого из оставшихся городов выходит, по крайней мере, 2 маршрута. Пусть есть город X, из которого выходит только 1 маршрут в город Y. Так как мы его не выкинули, значит до этого из него выходили маршруты в какие-либо из городов, которые мы выкинули. Z1, Z2, … Мы уже показали, что осталось не менее двух городов, значит, есть еще какой-то город, отличный от X и Y, но тогда из этого города нельзя было добраться ни да какого из городов Z1, Z2, … Таким образом, получаем, что из каждого из оставшихся городов выходит не менее двух маршрутов.

Следовательно, всего маршрутов не меньше, чем m. Но по предположению их m – 2. Возникшее противоречие доказывает, что количество маршрутов не может равняться п – 2.

Ответ: Б. n – 1.

Комментарий

С задачей опять справилось большинство, но все неверные ответы встречались в работах. В данном случае, можно было легко проверить, что варианты А, В и Г не верны для маленького n (например, для 3). Отметим так же, что вариант Г – это наибольшее количество автобусных маршрутов между двумя городами, это число получается, если каждый город соединить маршрутом с каждым.

Задача №8

Эскалатор стоящего на нём человека поднимает наверх за 80 с. Если человек спускается по движущемуся вниз эскалатору, начиная движение с верхней точки, шагая вниз и не пропуская ни одной ступеньки, то спуск займет 60 с, а человек пройдет 45 ступенек. Сколько ступенек между входом и сходом неподвижного эскалатора?

А. 200.

Б. 180.

В. 150.

Г. 120.

Решение

Обозначим искомое количество ступенек через х. За 80 с лента эскалатора сдвинулась на х ступенек, а за 60 с — на 60 ступенек. За это же время человек самостоятельно сдвинулся на 45 ступенек. Имеем уравнение: 60 + 45 = х или . Отсюда х = 180.

Ответ: Б. 180.

Комментарий

Больше половины участников с задачей справились. Часто встречались ответы В и Г, но как они были получены не понятно, потому что все ошибки при составлении уравнения проводили либо к тому, что не существует такого положительного x (кстати, встретилась работа, где школьник отмечает, что решения не существует), либо получался не целый x, которого не было среди возможных вариантов ответа.

Задача №9

Вода в некотором водоёме пополняется равномерно из источника. Известно, что 70 коров выпили бы воду из водоёма за 40 дней, а 50 коров — за 64 дня. Каков примерно объём воды в водоёме до запуска коров в этот водоём, если считать, что корова за день выпивает 80 л воды?

А. 170 м3.

Б. 17000 л.

В. 1700 л.

Г. 1700 м3.

Решение

Обозначим объём воды в водоёме до запуска коров в этот водоём через x (л), а через v л/день — скорость поступления воды из источника. Тогда из условия следует следующая система уравнений:

Умножив первое уравнение на 8, а второе — на 5 и вычтя почленно из первого уравнения второе, получим:

3х = 8070408 – 8050645 = 512 000 (л), х = 512 000:3  170 000 (л) = 170 (м3).

Ответ: А. 170 (м3).

Комментарий

Верных ответов в этой задаче было около половины. Путаница здесь могла возникнуть только при переводе литров в м3. Напомним, что 1 литр – это объем куба со стороной 1 дм (10 см), а 1 м3 – это объем куба со стороной 1 м, соответственно 1 м3 вмещает в себя 1000 кубов со стороной 1 дм (10*10*10).

Задача №10

В кинотеатре два прямоугольных зала для просмотра кинофильмов. В одном из них в каждом ряду по 38 мест, в другом – по 24 места, причём в первом зале на 168 мест больше, чем во втором. Сколько мест в двух залах вместе, если количество рядов в обоих залах более 20 и менее 40?

А. 1336.

Б. 1656.

В. 1800.

Г. 2112.

Решение

Обозначим количество рядов в зале, где 38 мест в каждом ряду, через х, а количество рядов в другом зале — через у. Тогда, по условию, 38х – 24у = 168, 20 < x < 40, 20 < y < 40. Из этого уравнения следует, что х делится на 4. Следовательно, х может принимать значения 24, 28, 32, 36. Для этих значений х полученное уравнение принимает вид:

12у = 1924 – 84 = 372, 12у = 1928 – 84 = 448,

12у = 1932 – 84 = 524, 12у = 1936 – 84 = 600.

Только первое и последнее уравнения имеют решения в целых числах. Но решение последнего не удовлетворяет условию у < 40. Следовательно, у = 31, х = 24 и в двух кинозалах 2438+ 3124 = 2469 = 1656 мест

Ответ: Б. 1656 мест.

Комментарий

И снова верных ответов было около половины. Задача была достаточно сложной, поэтому скорее всего неверные ответы указывали школьники наугад, не решив задачу.


9 класс

6

7

8

9

10

А

Б

А

А

Г

Задача №6

Во сколько раз длина пути, преодолеваемого концом минутной стрелки за 16 часов, больше длины пути, преодолеваемого концом часовой стрелки, которая в 1,5 раза короче минутной?

А. В 18 раз.

Б. В 16 раз.

В. В 9 раз.

Г. В 8 раз.

Решение

За 16 часов минутная стрелка сделала 16 оборотов. Следовательно, её конец пройдет путь, равный 162πR, где R — длина минутной стрелки. Часовая стрелка за 16 часов сделает один оборот и, кроме того, повернётся на 120°, или на  оборота. Следовательно, её конец пройдёт путь, равный R = 2πR. Искомое отношение равно 16: = 18.

Ответ: А. В 18 раз.

Комментарий

Задачу правильно решили около половины школьников. Все неверные ответы встречались в работах примерно с одинаковой частотой. Нашлась даже работа, в которой были указаны 2 ответа – А и Г, видимо школьник не прочитал в условии, какая из стрелок длиннее в 1,5 раза (Если бы длиннее была часовая стрелка, то верным был бы ответ Г). Возможно, те, кто выбирал ответ Г тоже читали условие не внимательно.

Задача №7

Робот начинает движение в некоторой точке, в начале движения он выбирает направление перемещения. Далее робот движется прямолинейно 10 м, затем поворачивает на 90° вправо или влево и движется прямолинейно 10 м, далее снова поворачивает на 90° вправо или влево и движется прямолинейно 10 м и т. д. Какое из приведенных чисел может быть приближённым расстоянием между началом и концом пути робота с точностью до 1 метра, если робот остановился на месте 6-го поворота?

А. 16 м.

Б. 31 м.

В. 44 м.

Г. 58 м.

Решение

Изобразим первое перемещение робота горизонтальным отрезком. Тогда дальнейшее движение робота будет проходить по сторонам квадратной сетки, изображённой на рисунке, со стороной 10 м. Точка О — начало движения.

Возможные варианты расположения робота на месте 6-го поворота указаны на рисунке. Возможные расстояния от начала движения до пункта на месте 6-го поворота могут равняться: ОА = 10  14 (м), ОВ =  31 (м), ОС = 30  42 (м).

Ответ: Б. 31 м.

Комментарий

С задачей справилось чуть более половины учеников. Самым распространенным среди неверных ответов был ответ В. Во всех вариантах для разных классов этой олимпиады были различные вариации про робота, который движется по линиям сетки, и везде были затруднения с тем, чтобы представить, как именно движется робот и нарисовать все возможные маршруты. В данном случае возможных маршрутов было 32 (на каждом из 5 поворотов у робота было 2 варианта), но легко было понять, что многие варианты одинаковы с точностью до симметрии.

Задача №8

Студенческую группу горного института назовём «женской», если в ней учится более двух девушек. Сравните процент р «женских» групп в институте с процентом q девушек, обучающихся в этих группах, если известно, что в институте есть как «женские», так и «не женские» группы.

А. p < q.

Б. p = q.

В. p > q.

Г. Сравнить нельзя.

Решение

Обозначим через х, у количество «женских» групп и количество «не женских» групп соответственно, а через а и с – количества девушек в «женских» и «не женских» группах. Тогда p = 100, q = 100. Из последнего равенства следует, что . Так как, по условию, количество девушек а в «женских» группах больше, чем 2х, а количество девушек с в «не женских» группах не больше 2у, то . Но тогда q > 100, то есть q > р.

Ответ: А. p < q.

Комментарий

Эта задача оказалась самой сложной в первой части олимпиады. Верно ответили всего около четверти учеников. Самый распространенный ответ в работах – Г, его выбрали около половины участников. Еще раз напомним, что ответ «сравнить нельзя» будет верным только в том случае, если существуют такие варианты разбивок студентов на группы, при которых может быть больше как p, так и q, а не в том случае, если школьнику не удалось придумать способа решить задачу.

Задача №9

В старшей группе детского сада, состоящей из 25 детей, раздали 100 конфет. Оказалось, что у мальчиков столько же конфет, сколько и у девочек. В группе нет мальчиков, у которых конфет больше, чем у Пети, и нет девочек, у которых конфет больше, чем у Маши. Какое количество конфет из приведенных не могло быть у Пети и Маши вместе?

А. 7.

Б. 10.

В. 14.

Г. 21.

Решение

Введём следующие обозначения: m — количество мальчиков в группе, d — количество девочек в группе, x — количество конфет у Пети, y — количество конфет у Маши. По условию, xm  50, yd  50. Оценим общее количество конфет у Пети и Маши.

х + у  .

Так как произведение m(25 – m) принимает наибольшее значение при равенстве сомножителей, то х + у   = 8. Следовательно, у Пети и Маши вместе из приведенных не могло быть 7 конфет.

Ответ: А. 7.

Комментарий

Эту задачу решило почти две третьих участников. В приведенном решении дается оценка снизу, какое минимальное количество конфет должно быть у Пети и Маши, из которой сразу следует, что верный ответ А, но стоит отметить, что если бы эта задача была без вариантов ответа, требовалось бы не только доказать, что у детей не могло быть ровно 7 конфет, но и привести примеры, либо по другому обосновать, что все другие количества конфет у них могли быть.

Задача №10

Расстояние между городами Х и Y равно 240 км. В 8 часов утра из города Х выехал автобус, прибывший в город Y в 13 часов. Каждый раз через один и тот же промежуток времени автобус делал остановку длительностью 15 мин. Всего автобус сделал 4 остановки. Одновременно с автобусом из города Х в город Y выехало такси, которое, возвращаясь обратно после доставки пассажиров в город Y, встретило автобус на расстоянии 90 км от города Y. Какова примерно средняя скорость движения такси, если считать, что автобус и такси двигались практически равномерно? Выберите наиболее точное значение. .

А. 90 км/ч.

Б. 94 км/ч.

В. 98 км/ч.

Г. 102 км/ч.

Решение

Пусть А, В, С, D — обозначения остановок на пути из Х в Y (см. рис.). Из условия следует, что средняя скорость движения автобуса равна 240:4 = 60 (км/ч) и ХА = АВ = ВС = СD = DY =  = 48 (км).

Встреча автобуса и такси произошла на расстоянии 90 км от города Y, то есть между С и D. К моменту встречи такси проехало 240 + 90 = 330 (км). От момента выезда автобуса до момента встречи с такси прошло (240 – 90): 60 +  = (ч). Средняя скорость такси равна 330:  102 (км/ч).

Ответ: Г. 102 км/ч.

Комментарий

Эту задачу верно решили около половины учеников. Самым распространенным среди неправильных ответов был вариант А, но скорее всего этот ответ был получен наугад.




Предварительный просмотр:

Error! Reference source not found.


Разбор задач пятой части заданий

  1. 4-5 класс

Задача №6.

Число, выражающее площадь прямоугольной комнаты в м2, на единицу больше числа, выражающего периметр этой комнаты в м. Каковы размеры комнаты, если её длина и ширина выражаются целыми числами метров?

Решение 1:

Предположим, что прямоугольник на рисунке справа является изображением комнаты. Его площадь равна произведению длин его сторон, а периметр – их удвоенной сумме. Следовательно периметр выражается чётным числом метров. Но, по условию, число, выражающее площадь комнаты в м2, на единицу больше числа, выражающего периметр этой комнаты в м. Следовательно, число, выражающее площадь, нечётно, а отсюда следует, что и длины сторон выражаются нечётными числами. Учитывая, что длина и ширина комнаты не могут равняться 1 м, а, следовательно, они не менее 3 м, осталось проверить, для каких из следующих чисел 3, 5, 7, 9, … произведение на 1 больше удвоенной суммы. Проверку выполним с помощью таблицы.

Длина

3

3

3

3

3

5

5

5

5

7

7

7

9

9

Ширина

3

5

7

9

11

5

7

9

11

7

9

11

9

11

Периметр

12

16

20

24

28

20

24

28

32

28

32

36

36

40

Площадь

9

15

21

27

33

25

35

45

55

49

63

77

81

99

Из таблицы видно, что число, выражающее площадь, на единицу больше числа, выражающего периметр, если комната имеет размеры 3×7.

Ответ. 3×7 м.

Комментарий

В одной из работ было приведено очень оригинальное решение, с которым я хотел бы вас ознакомить.

Для начала убедимся в том, что стороны прямоугольника имеют длину не меньше 3. Так как длина и ширина комнаты выражаются целыми числами, то периметр комнаты будет четным числом. Мы знаем, что площадь комнаты на единицу больше периметра значит, площадь выражается нечетным числом. Получаем, что длина и ширина комнаты – нечетные числа. Пусть размеры комнаты m и n. Из условия мы знаем, что m*n=2*(m+n)+1. Если одна из сторон равна 1, то m*1=2*(m+1)+1, то есть m=2*m+1. Данное уравнение не имеет решения среди целых положительных чисел. Таким образом, мы получили, что каждая сторона прямоугольника имеет длину не меньше 3. Рассмотрим прямоугольник со сторонами n-2 и m-2. Его площадь равна (n-2)*(m-2)=n*m-2*n-2*m+4. Подставив вместо m*n выражение  2*(m+n)+1 (они равны по условию) получим, что площадь прямоугольника со сторонами n-2 и m-2 равна 2*(m+n)+1-2*n-2*m+4=5. Так как 5 является простым числом, то стороны такого прямоугольника равны 1 и 5. А значит, стороны искомого прямоугольника равны 3 и 7.

Многие участники конкурса дали правильный ответ в этой задаче, но не дали никаких пояснений. Естественно они не получили за это много баллов. Если вы решаете задачу перебором, приводите в решении проделанный перебор, и лучше, если он будет полный, ведь вы не доказываете отсутствие ответов среди нерассмотренных случаев. Многие почему-то дали ответ 3 и 6, видимо посчитав, что периметр должен быть равен площади. Некоторые дали ответ 3 и 5, видимо посчитав, что периметр на единицу больше площади. Внимательно читайте условие. Читайте его несколько раз. Проверяйте, удовлетворяют ли ваши ответы условию. Это избавит вас от настолько глупых ошибок.

Задача №7.

Восемь мальчиков выстроились по кругу. Если обходить их, двигаясь по часовой стрелке, то окажется, что:

А мы встретим раньше Б и В,

Б – после К через одного;

Л – раньше А, но после Д;

В – после М через одного;

Д – раньше Б;

В – раньше, чем Г;

М – после К.

Кого мы встретим первым и кого последним?

Решение:

На рис. 1 изображено расположение мальчиков. Если обход начать в любой точке дуги ГД, то убедимся, что выполняются все условия. Но если начать обход из любой другой точки окружности, обязательно обнаружим нарушение каких-то условий. Следовательно, первым встретим Д, а последним – Г.

Покажем, что другого расположения мальчиков по кругу, удовлетворяющего всем условиям, не существует. Пусть Б находится не между В и М. Тогда, двигаясь по часовой стрелке, мы встречаем В после М через одного, М – после К и Б – после К через одного (см. рис. 2). Так как А встречаем раньше Б, Д и Л – раньше А, а В – раньше Г, то между В и М некого поставить. Пришли к противоречию. Следовательно, Б стоит между В и М. Из условий: Л – раньше А, но после Д, А мы встретим раньше Б и В, В – раньше, чем Г получаем размещение, изображённое на рис. 1.

Ответ. Первым – Д, последним – Г.

Комментарий

Очень многие дали правильный ответ в этой задаче. Но лишь единицы из них попытались доказать, что такое расположение единственное. Хотя это надо было сделать обязательно. Ведь если бы задача имела несколько решений, то найдя одно из них нельзя утверждать, что мальчики располагались именно так.

Задача №8.

За круглым столом вам нужно посадить 10 гостей, среди которых 5 мальчиков и 5 девочек. Можно ли рассадить их так, чтобы ни у одного гостя обоими соседями не были мальчики?

Решение 1:

Предположим, что требуемое рассаживание возможно. Обозначим через Х1, Х2, Х3, …, Х10 гостей, сидящих в указанном порядке за круглым столом при движении по часовой стрелке (см. рис.). Обозначения можно выбрать так, что Х1 – девочка, а Х2 – мальчик. Этот выбор не ограничивает общности рассуждений. Тогда Х10 – обязательно девочка, ибо в противном случае у Х1 оба соседи будут мальчики. Кто бы ни был Х3, Х4 – девочка.

Если Х3 – мальчик, то Х5 – девочка. Но тогда среди гостей Х6, Х7, Х8, Х9 – одна девочка, и её соседями являются мальчики.

Если Х3 – девочка, то среди гостей Х5, Х6, Х7, Х8, Х9 только одна девочка (всего за столом 5 девочек) и в любом случае соседями одного из гостей будут мальчики. Снова получено противоречие.

Ответ. Нельзя.

Решение 2:

Для начала пронумеруем места за столом от 1 до 10. Рассмотрим две группы людей, первая – сидящие на четных местах, вторая – на нечетных местах. Так как мальчиков по условия 5, то в одной из групп их будет не меньше чем 3. Чтобы соблюдалось условие задачи, необходимо чтобы у каждого мальчика ближайшими соседями из той же группы что и он были девочки. Но так как в одной из групп мальчиков больше чем девочек, то это невозможно.

Очень просто объяснить почему это так. Пусть за круглым столом сидят 5 человек, среди которых есть 3 мальчика. Пронумеруем места по порядку от 1 до 5 (допустим по часовой стрелке), начиная с любого мальчика. Так как на месте номер 1 сидит мальчик, то на местах с номерами 2 и 5 должны сидеть девочки. Но так как количество мальчиков не меньше 3, то на местах 3 и 4 сидят мальчики.

Таким образом мы доказали что есть мальчик у которого ближайший сосед той же группы (слева либо справа) будет тоже мальчик. Значит у человека из другой группы, сидящего между ними оба соседа мальчики.

Ответ: нельзя.

Решение 3:

Назовем «группой мальчиков(девочек)» мальчиков(девочек) сидящих подряд при условии того что справа и слева от них сидят девочки(мальчики). Заметим, что в каждой группе мальчиков не может быть более двух человек, иначе в этой группе будет мальчик, соседи которого тоже мальчики. Также заметим, что в  каждой группе девочек не менее двух девочек, иначе, если она одна, то ее соседи мальчики. Итак, раз по условию 5 девочек, то они могут составлять не более двух групп девочек. В то время как, групп мальчиков не может быть менее трех (так как мальчиков всего 5, а в каждой группе не более двух). После каждой группы девочек должна идти группа мальчиков и наоборот. Но так как групп девочек меньше чем групп мальчиков то это невозможно. Получили противоречие.

Ответ: нельзя.

Комментарий

Практически у всех участников решение данной задачи ограничивалось словом «да» либо «нет». Некоторые еще приводили 1 или 2 примера. Мало кто смог привести нормальное решение. Хотя способов решения этой задачи довольно много.

Учитесь правильно выражать свои мысли. Уделите решению больше времени. При необходимости введите свою терминологию, но дайте четкие определения. Это не только поможет вам получить большее количество баллов на конкурсах, но и также поможет грамотно общаться с людьми, излагать свои мысли так, чтобы окружающие вас понимали и не задавали лишних вопросов.

Задача №9.

Сад имеет форму квадрата со стороной 100 м. Расстояние между любыми двумя деревьями в саду не меньше 10 м. Все деревья растут на расстоянии не менее 5 м от ограды в рядах, параллельных ограде. Какое наибольшее число деревьев может быть в саду?

Комментарий

Данная задача вызвала много вопросов. В условии не было объяснено, что значит «в рядах, параллельных ограде». Отсюда возникло множество разных, но, казалось бы, правильных ответов. Оригинал решения выглядел так:

Решение 1:

Пусть квадрат со стороной 100 м на рисунке справа является изображением сада. Тогда точки, удалённые от сторон квадрата на 5 м, образуют квадрат со стороной 90 м. Если разбить стороны меньшего квадрата на 9 равных частей и через точки деления провести отрезки, параллельные его сторонам, то точки пересечения этих отрезков между собой и со сторонами квадрата, а также 4 вершины квадрата, могут служить изображениями мест посадки деревьев. Их 100. Это наибольшее число деревьев, которое может быть посажено в саду.

Ответ. 100.

Комментарий

Также имело место решение в котором деревья располагались несколько иначе.

Решение 2:

Для расположения деревьев мы имеем квадрат 90х90.

Если располагать деревья так, как показано на рисунке 1, то можно вместить большее количество деревьев. Таким образом, мы сможем разместить 7 рядов по 10 деревьев и 4 ряда по 9 деревьев (как показано на рисунке 2). Расстояние между рядами с 10 деревьями и с 9 деревьями равно 5√3. Таких промежутков 8 и их сумма меньше 70. Значит, на оставшемся месте можно разместить 2 ряда по 10 деревьев. 7*10+4*9=106.

Ответ: 106.

Комментарий

В данной задаче ставился максимальный балл за любой пример, который удовлетворял условию и включал в себя не менее 100 деревьев. Большинство участников смогли построить такой пример.

По-хорошему, решение данной задачи должно включать в себя доказательство того, что большее количество деревьев нельзя расположить так, чтобы все условия соблюдались. Но это задача точно не для 4-5 классов, поэтому мы решили опустить этот момент. Доказательство этого факта я не буду приводить, так как оно содержит формулы из геометрии, с которыми, думаю, вы еще не знакомы.

Задача №10.

Сколько существует различных квадратов, длины сторон которых выражаются целыми числами сантиметров, площади которых не превышают 40 см2, которые можно разрезать на части, равные фигуре, изображённой на рисунке и состоящей из четырёх квадратов со стороной 1 см?

Решение 1:

Существует 6 квадратов, длины сторон которых выражаются целыми числами сантиметров, площади которых не превышают 40 см2. Длины их сторон равны 1 см, 2 см, 3 см, 4 см, 5 см, 6 см. Они состоят из 1, 4, 9, 16, 25, 36 квадратных единиц, то есть квадратиков со стороной 1 см.

Если квадрат можно разрезать на равные части, то его площадь должна нацело делиться на площадь, в данном случае на 4. Это требование выполнено для трёх квадратов: 2×2, 4×4, 6×6.

Квадрат 2×2, очевидно, требованиям задачи не удовлетворяет.

Разрезание квадрата 4×4 на части, равные данной фигуре, показано на рис. 1.

Квадрат 6×6 нельзя разрезать на равные части, равные данной фигуре. Установить это можно, начав покрытие квадрата 6×6 фигурами указанного вида. Без ограничения общности можно считать, что одна плитка лежит в левом верхнем углу, как показано на рис. 2. Тогда расположение плитки 2 однозначно. Но тогда нижний левый угол квадрата покрыть данными фигурами невозможно.

Ответ. 1.

Решение 2:

Так как площадь квадратов со стороной больше 6 см превышает 40 см2, то искомые квадраты имеют сторону не больше 6 см. Площадь фигуры 4 см2, значит площади искомых квадратов должны делиться на 4. Таким образом, отсеиваются квадраты со стороной 1 см, 3 см, и 5 см. Рассмотрим квадраты со сторонами 2 см, 4 см и 6 см. Выполним в них шахматную раскраску. Заметим, что при любом расположении данной фигуры из четырех клеток, она покрывает 3 клетки одного цвета и одну клетку другого. Назовем фигуру четной, если она покрывает 3 черных клетки и 1 белую, нечетной – если 1 черную клетку и 3 белых. Так как в квадратах со стороной 2 см, 4см и 6 см количества черных и белых клеток равны между собой, то количества четных и нечетных фигур в покрытии должны совпадать. Тогда количество покрытых клеток одного цвета должно делиться на 4 (так как четная и нечетная фигуры покрывают вместе по 4 клетки каждого цвета). Количество черных клеток в квадратах со стороной 2 см и 6 см не делится на 4. Таким образом, остается только квадрат со стороной 4 см. Для него строится пример (смотрим решение 1).

Комментарий

Большинство участников построили пример покрытия квадрата 4х4, но не доказали, что остальные квадраты не подходят. Может быть не так уж и просто доказать, что квадрат 6х6 не имеет такого покрытия фигурками, но с остальными квадратами не должно возникать проблем. Если бы эти участники объяснили, почему не подходят квадраты со сторонами 1, 2, 3 или 5, то их итоговый результат стал бы выше. Так что советую вам приводить доказательства в тех случаях, в которых вы можете. Это повлияет на ваш результат только в положительную сторону.


  1. 6-7 класс

Задача №6.

Среди задач конкурсного задания по математике есть алгебраические и геометрические. Среди них есть трудные и лёгкие. Можно ли среди них выбрать две такие задачи, которые были бы из разных разделов математики (из алгебры и геометрии) и разной трудности?

Решение:

Рассмотрим совокупности алгебраических и геометрических задач. Если они обе состоят из задач одинаковой трудности, то достаточно взять из каждой совокупности по одной задаче, и мы получим искомый набор двух задач.

Если, например, среди алгебраических задач есть задачи различной трудности, то, взяв любую геометрическую задачу, можно выбрать алгебраическую задачу иной трудности.

Ответ. Можно.

Комментарий

Задача довольно простая. Но, тем не менее, многие участники сочли достаточным привести только ответ «да». А как же доказательство? Не игнорируйте доказательства там, где задание требует его привести. Тем более в простых задачах. Ответы без доказательства ничего не стоят, особенно в задачах, в которых спрашивается «да или нет?».

Задача №7.

На четырёх стенах комнаты и на её потолке нужно наклеить различное количество снежинок так, чтобы на каждой стене была хотя бы одна снежинка, но не более 7, а суммы количеств снежинок на противоположных стенах были равны и равнялись числу снежинок на потолке. Сколько существует различных вариантов выполнения этого задания, если различные варианты отличаются числом снежинок хотя бы на одной стене?

Решение:

Если обозначить через (а1, b1), (а2, b2) количества снежинок на противоположных стенах, то, по условию, выполняются соотношения

а1 + b1 = а2 + b2, 1  а1  7, 1  а2  7, 1  b1  7, 1  b2  7.

Количество снежинок на потолке равно сумме а1 + b1. Сумма а1 + b1 может принимать значения от 5 до 11. Числа 3, 4, 12 и 13 можно представить в виде суммы различных натуральных чисел, не больших 7, только одним способом. Следовательно, в этих случаях нельзя выбрать две указанные пары.

Пусть а1 + b1 = 11. Так как 11 = 7 + 4 = 6 + 5 и других вариантов записи числа 11 в виде суммы двух различных натуральных чисел, не больших 7, нет, то в этом случае существует один вариант выбора двух пар чисел, удовлетворяющих условию: (7, 4), (6, 5).

Пусть а1 + b1 = 10. Так как 10 = 7 + 3 = 6 + 4, то в этом случае существует один вариант выбора двух пар чисел, удовлетворяющих условию: (7, 3), (6, 4).

Пусть а1 + b1 = 9. Так как 9 = 7 + 2 = 6 + 3 = 5 + 4, то в этом случае существует три варианта выбора двух пар чисел, удовлетворяющих условию: (7, 2), (6, 3), (5, 4).

Пусть а1 + b1 = 8. Так как 8 = 7 + 1 = 6 + 2 = 5 + 3, то в этом случае существует три варианта выбора двух пар чисел, удовлетворяющих условию: (7, 1), (6, 2), (5, 3).

Пусть а1 + b1 = 7. Так как 7 = 6 + 1 = 5 + 2 = 4 + 3, то в этом случае существует три варианта выбора двух пар чисел, удовлетворяющих условию: (6, 1), (5, 2), (4, 3).

Пусть а1 + b1 = 6. Так как 6 = 5 + 1 = 4 + 2, то в этом случае существует один вариант выбора двух пар чисел, удовлетворяющих условию: (5, 1), (4, 2).

Пусть а1 + b1 = 5. Так как 5 = 4 + 1 = 3 + 2, то в этом случае существует один вариант выбора двух пар чисел, удовлетворяющих условию: (4, 1), (3, 2).

Всего 13 вариантов. Каждый вариант приводит к восьми вариантам расклейки снежинок на стенах. Например, вариант (6, 2) и (7, 1) приводит к следующим вариантам:

Всего имеем 138 = 104 варианта выполнения задания.

Комментарий

Задача не сложная. Здесь только нужно внимательно посчитать  все возможные варианты. Большинство участников с этим заданием справились. Хотя опять же почему-то не сочли нужным приводить свой способ решения.

Задача №8.

Спецотряду из двенадцати солдат нужно одновременно добраться в пункт назначения, находящийся в 40 км от их места расположения. В их распоряжении есть только автомобиль, которым управляет водитель, не входящий в этот спецотряд, который вмещает 4 солдат и движется со скоростью 40 км/ч. Скорость движения солдат пешком равна 8 км/ч. Какое наименьшее время понадобится для выполнения задания?

Решение 1:

Можно предложить такой план действий.

1) В течение 36 мин машина перевозит 4-х солдат на 24 км, остальные 8 солдат за это время проходят расстояние, равное  км.

2) Расстояние между машиной и 8-ю солдатами составляет 24 – 4,8 = 19,2 км. Машина выезжает навстречу 8-и солдатам, скорость их сближения составляет 48 км/ч. Они встретятся через 19,2:48 = 0,4 (ч) или через 24 мин. Четверо перевезенных солдат идут к месту назначения.

3) Машина перевозит 4-х из 8-и солдат в направлении к месту назначения в течение 36 мин, остальные движутся пешком к месту назначения.

4) Расстояние между машиной и 4-мя солдатами составляет 24 – 4,8 = 19,2 км. Машина выезжает навстречу 4-м солдатам, скорость их сближения составляет 48 км/ч. Они встретятся через 19,2:48 = 0,4 (ч) или через 24 мин. Восемь перевезенных солдат идут к месту назначения.

5) Машина перевозит 4-х последних солдат к месту назначения в течение 36 мин, остальные движутся пешком к месту назначения и через те же 36 мин все окажутся у места назначения.

Всего для выполнения задания понадобилось 36 мин + 24 мин + 36 мин + 24 мин + 36 мин = 2 ч 36 мин. Это время минимально, так как все всё время двигались с наибольшей возможной скоростью и машина использовалась полностью, в направлении к месту назначения она всё время была полностью заполненной.

Ответ. 2 ч 36 мин.

Комментарий

Конечно, это хорошо строить свое решение на том, что вы уже знаете оптимальный план действий. Но как же поступить, если он вам не известен.

Решение 2:

Для начала надо понять, что наименьшее время получится, если машина будет использоваться полностью и все 12 солдат попадут в пункт назначения одновременно. Далее, поймем, что машина подбросила каждого из солдат на одинаковое расстояние. Это действительно так. Иначе солдаты прибудут в пункт назначения не одновременно, и время не будет минимальным. Также заметим, что для того, чтобы подвезти каждого солдата машина сделала 3 рейса в сторону пункта назначения (загруженная) и 2 рейса в обратную сторону (пустая). Теперь надо найти, на какое расстояние машина подбросила каждого из солдат. Возьмем искомое расстояние за x и составим уравнение.

(40-x)/8+x/40=3*x/40+2*(x-(x/40)*8)/(40+8)

В левой части уравнения – время, за которое один солдат добирается до пункта назначения. Время, которое он шел пешком равно (40-x)/8, а время, которое он ехал на машине равно x/40.

В правой части уравнения – время использования машины. Три рейса в сторону пункта назначения заняли 3*x/40 времени, а два рейса в обратную сторону заняли 2*(x-(x/40)*8)/(40+8) времени.

Левая часть уравнения должна быть равна правой части, так как мы считаем, что машина использовалась полностью.

Приведя все к общему знаменателю в обеих частях получаем:

(200-5x+x)/40=(18x+10x-2x)/240, отсюда получаем, что

1200-24x=26x,

x=24 (км).

Теперь остается только найти затраченное время, подставив x=24 в левую часть уравнения.

(40-24)/8+24/40=16/8+24/40=2+0,6=2,6. В часах это равняется 2 часа 36 минут.

Комментарий

Данная задача довольно сложная, хотя многие участники смогли найти правильный ответ. Конечно, не все они объяснили, почему полученное ими время будет наименьшим, но здесь за это они потеряли не так много баллов.

Не забывайте доказывать, что ваш ответ наименьший (или наибольший) когда этого требует задача.

Задача №9.

Некоторая компания собиралась 5 раз. Каждый раз присутствовали 4 человека из этой компании, причём никакие двое из её членов не были на встрече вместе более одного раза. Сколько человек может быть в такой компании, если каждый её член был хотя бы раз на сборе?

Решение:

Покажем, что обеспечить выполнение условий задания может только компания, в которой не менее 10 человек и не более 20. Если каждый член компании присутствовал на встрече только 1 раз, то в компании 20 человек. Если какой-то человек был на сборах хотя бы 3 раза, то на этих трех встречах с ним побывали еще 9 человек, так как никакие двое не встречались 2 раза. Осталось рассмотреть случай, когда ни один человек не был на встрече 3 раза и есть человек, который был 2 раза. Назовем а1 – человека который был на двух встречах. Пусть на первой встрече были люди а1, а2, а3 и а4. Тогда на второй встрече а1, а5, а6 и а7. На каждой их оставшихся трех встреч могли присутствовать не более чем по одному человеку с каждой из этих встреч. Таким образом, если в компании будет менее 10 человек, то будут люди, которые встречались между собой более одного раза.

Пример для 10 человек:

а1  а2  а3  а4 – первая встреча.

а1  а5  а6  а7 – вторая встреча.

а2  а5  а8  а9 – третья встреча.

а3  а6  а8  а10 – четвертая встреча.

а4  а7  а9  а10 – пятая встреча.

Примеры для компаний из 11-19 человек строятся просто заменой на одной из встреч человека, который был еще на какой-либо встрече, на нового члена компании.

Ответ: в компании не менее 10 и не более 20 человек.

Комментарий

Задача довольно простая. Но многие участники конкурса, найдя минимальное количество людей в компании на этом остановились. В условии же сказано, что каждый был хоть раз на сборе. Это сделано для того чтобы была верхняя граница количества человек в компании. Ее тоже необходимо было найти. Будьте внимательней. Не бросайте решение на половине пути.

Задача №10.

Нужно разложить 45 яблок по k коробкам, расположенным по окружности, так, чтобы в любых двух соседних коробках число яблок отличалось друг от друга на 1 яблоко. Каким может быть число k, если в каждой коробке должно лежать хотя бы одно яблоко?

Решение

Если разложение 45 яблок по корзинам, удовлетворяющее условиям задания, возможно, то число k должно быть чётным, не делящимся на 4. Действительно, в каждой паре соседних коробок находится нечётное число яблок, число таких пар нечётно, так как 45 – число нечётное. Следовательно, число коробок чётно, но не делится на 4.

Число коробок не может быть больше 30, так как в каждой паре соседних коробок не менее трёх яблок и, следовательно, пар не больше, чем 45:3 = 15.

Осталось показать, что число коробок может равняться 2, 6, 10, 14, 18, 22, 26, 30. Это сделано в следующей таблице

K

Распределение яблок по коробкам

2

22, 23

6

7, 8, 7, 8, 7, 8

10

4, 5, 4, 5, 4, 5, 4, 5, 4, 5

14

3, 4, 3, 4, 3, 4, 3, 4, 3, 4, 3, 2, 3, 2

18

3, 2, 3, 2, 3, 2, 3, 2, 3, 2, 3, 2, 3, 2, 3, 2, 3, 2

22

3, 2, 3, 2, 3, 2, 3, 2, 3, 2, 3, 2, 1, 2, 1, 2, 1, 2, 1, 2, 1, 2

26

3, 2, 3, 2, 3, 2, 1, 2, 1, 2, 1, 2, 1, 2, 1, 2, 1, 2, 1, 2, 1, 2, 1, 2, 1, 2

30

1, 2, 1, 2, 1, 2, 1, 2, 1, 2, 1, 2, 1, 2, 1, 2, 1, 2, 1, 2, 1, 2, 1, 2, 1, 2, 1, 2, 1, 2

Ответ. 2, 6, 10, 14, 18, 22, 26, 30.

Комментарий

Задача решается простым перебором, но прежде чем выполнять перебор надо заметить один факт который этот перебор значительно сокращает. Хотя находились и те, кто перебрал все варианты, а не только четные не делящиеся на 4.


  1. 8-9 класс

Задача №6.

Можно ли разрезать правильный шестиугольник прямолинейными разрезами так, чтобы из полученных частей можно было сложить без наложений ромб?

Решение:

Решение см. на рисунке. Возможность складывания из треугольников 2, 3, 4 треугольника следует из того, что углы при вершинах правильного шестиугольника равны по 120°, а диагонали, соединяющие две вершины, между которыми находится ровно одна вершина, равны между собой.

Ответ. Можно.

Комментарий

Задача довольно простая. И практически все участники конкурса дали здесь правильный ответ. Некоторые даже привели различные объяснения, почему получившаяся фигура является ромбом. Многие нарисовали рисунок. Но, к сожалению, было огромное количество тех, кто посчитал достаточным просто написать слово «можно». Эти ученики получили всего по одному баллу. Олимпиада по математике это не лотерея где надо правильно ответить на вопрос «да» или «нет». Необходимо обосновывать свои ответы. Не забывайте, пожалуйста, это делать.

Задача №7.

Прямоугольный стол, размером 1м на 2м, заставлен одинаковыми круглыми тарелками, диаметр которых равен 20 см. Какой наибольший диаметр может иметь свеча, которую можно наверняка поставить на стол, не передвигая тарелок, если диаметры свечей выражаются целым числом сантиметров?

Решение:

Очевидно, что нужно рассматривать самый неблагоприятный случай, когда тарелки стоят вплотную. Задача  сводится к нахождению наибольшего диаметра кружочка, который можно поместить между кругами одинакового диаметра, касающимися друг друга (см. рис. 1 и рис. 2). Очевидно, что в случае трёх кругов (рис. 2) рассматриваемая величина меньше, чем в случае четырёх (рис. 1), а тем более пяти, шести и более кругов.

Так как тарелки могли на середине стола касаться друг друга так, как показано на рис. 2, то больше, чем в этом случае, диаметр свечи быть не может.

Обозначим через О центр круга, который касается кругов с центрами О1, О2, О3, а через r – его радиус (рис. 2). Из условия касания маленького кружочка и больших кругов следует, что О – центр равностороннего треугольника О1О2О3 со стороной 20 см. Тогда ОО1 = ОО2 = ОО3 = r + 10, а с другой стороны ОО1 = . Следовательно, , а . Отсюда 2r   3,1, то есть искомый диаметр равен 3 см.

Ответ. 3 см.

Комментарий

Очередная задача по геометрии не требующая особых знаний. У многих учеников она не вызвала никаких затруднений. Хотелось бы только еще раз повторить, что надо внимательнее читать условие. Так как далеко не все обратили внимание на то, что диаметр должен быть целым числом.

Задача №8.

Квадрат разбит четырьмя отрезками на 9 квадратиков, как это показано на рисунке. Сколько различных развёрток поверхности куба можно получить из этого квадрата, если можно делать разрезы только по сторонам квадратиков и склеивать два квадратика, перегибая их по сторонам?

Решение:

Исходя из условий разрезания, можно сделать вывод, что развёртка поверхности куба должна состоять из 6 квадратов, указанных на рисунке. Следовательно, для её получения нужно три пары квадратиков склеить. На рис. 1, 2, 3 и 4 показаны четыре способа получения развёртки поверхности куба. На них склеенные квадратики закрашены.    

Других развёрток, не равных указанным, то есть таких, которые нельзя получить из указанных с помощью перемещений (параллельных переносов, поворотов, симметрий и др.) не существует.

Рассмотрим сначала получение развёрток, у которых центральная клетка на рис. 6 не перемещалась с помощью разрезания. Без ограничения общности, для удобства можно считать, что она является нижней гранью поверхности кубика на рис. 5. В этом случае, чтобы иметь верхнюю грань, нужно сделать разрез и склеивание такого вида, как это показано на рис. 7. Таких преобразований 8 (у каждой вершины 2). Все они – равные фигуры, Поэтому достаточно проанализировать варианты получения развёрток из фигуры на рис. 7.

Квадрат 4 обязательно является боковой гранью кубика. При этом есть всего четыре варианта выбора боковых граней, противоположных квадратам 2 и 4 (см. рис. 8 – 11).

Если же центральная клетка на рис. 6 вырезалась, то возможные варианты её склеивания с другими показаны на рис. 12 – 15.

Из указанных фигур склеиванием двух квадратиков получить развёртку поверхности куба нельзя.

Ответ. 4.

Комментарий

Хорошая задачка для рассуждения. Единственный ее минус состоит в том, что изложить подробное и строгое решение на бумаге не так-то просто. Наверное, именно поэтому многие решили этого не делать.

Задача №9.

К обменному пункту подошёл человек, подал 5 000 руб. и попросил дать ему целое число украинских гривен, целое число американских долларов и целое число евро, причём долларов в 10 раз меньше, чем гривен, а евро – минимально возможную сумму на остальные деньги. Сколько гривен, долларов, евро он получил, если за 4 руб. давали 1 грн., за 30 руб. – 1 доллар и за 40 руб. – 1 евро и все его деньги были использованы?

Решение:

Пусть клиент обменного пункта получил х долларов, за них с него взяли 30х руб. Из условия следует, что ему дали 10х грн., вычтя из поданной им суммы 40х руб. На оставшиеся деньги, то есть на 5 000 – 70х ему дали евро. Так как, по условию требуется, чтобы сумма в евро была наименьшей возможной, и, учитывая, что за 1 евро берут 40 руб., делаем вывод, что требуется найти такое значение х, чтобы при этом значении выражение 5 000 – 70х делилось на 40 и частное было наименьшим из всех возможных. Для этого нужно, чтобы х делилось на 4 и принимало из всех допустимых значений х наибольшее значение. Нетрудно видеть, что это значение х равно 68, то есть клиент получил 68 долларов, 680 гривен и  евро.

Ответ. 68 долларов, 680 гривен и 6 евро.

Комментарий

Многие участники конкурса без особого труда обнаружили, что количество долларов должно делиться на 4. А далее привели небольшой перебор, показавший, что искомое количество долларов равно 68.

Задача №10.

Два велосипедиста выехали из пунктов А и В навстречу друг другу и встретились в 5 км от А. Каждый из них, доехав до другого пункта, незамедлительно возвращался назад в пункт, из которого он выехал, и вторая их встреча произошла в 3 км от В. Каково расстояние между А и В?

Решение:

Обозначим через х и у скорости движения велосипедистов, выехавших, соответственно из А и В. Пусть первая встреча произошла в точке С, а вторая – в точке D (см. рис.). В следующей таблице представлены соотношения между расстояниями, скоростями, временами движения обоих велосипедистов на различных участках пути.

Участок движения

Путь, км

Скорость, км/ч

Время, ч

Движение І на АС

5

x

Движение ІІ на ВС

y

Движение І на СВ

x

Движение ІІ на СА

5

y

Движение І на ВD

3

x

Движение ІІ на АD

y

Учитывая, что между двумя встречами велосипедисты двигались одинаковое время, имеем уравнение: . После несложных преобразований получим: 5у2 – 7х2 – 2ху = 0 или 5t2 – 2t – 7 = 0, где . Решая квадратное уравнение и учитывая, что t > 0, получим: . Расстояние между А и В равно  + 5 (см. 2-ю строку таблицы). Поэтому АВ = 12 км.

Ответ. 12 км.

Комментарий

На самом деле, решение задачи намного сложнее, чем показано выше. Так как задача имеет несколько различных вариантов развития событий. Ведь в условии не сказано, что вторая встреча велосипедистов произошла тогда, когда  они оба возвращались в тот пункт, из которого выехали изначально. Могло получиться и так, что велосипедист, выехавший из А не доехал до пункта В, к тому моменту, когда второй побывав в пункте А догнал первого на обратном пути. И так же есть вариант, при котором вторая встреча произошла раньше, чем второй доехал до пункта А. Оба  эти варианта не противоречат условию. Решения в этих случаях строятся по тому же принципу, но имеют гораздо более некрасивые ответы, поэтому я не буду вам их приводить. В качестве упражнения можете попробовать сделать это самостоятельно.

Подробное решение хотя бы одного возможного случая оценивалось здесь в полный балл. Но нашлись и такие работы, в которых были рассмотрены несколько случаев, что не может ни радовать.




Предварительный просмотр:

Error! Reference source not found.


Разбор задач четвертой части заданий

  1. 4-5 класс

Задача №1.

В шкатулке катушки с белыми, чёрными, коричневыми и зелёными нитками. Известно, что катушек с белыми нитками в два раза больше, чем с чёрными, а катушек с чёрными нитками вдвое больше, чем с зелёными. Число катушек с коричневыми нитками меньше 7. Сколько катушек с нитками каждого цвета лежит в шкатулке. Если всего там 27 катушек?

Решение1:

Из условия следует, что всего в шкатулке катушек с белыми, чёрными и зелёными нитками больше 20. Примем число катушек с зелёными нитками за 1 часть, тогда число катушек с чёрными нитками составит 2 части, а с белыми – 4 части. Число катушек с нитками указанных трёх цветов составляет 7 частей. Так как их более 20, но не более 27, то их число равно 21 (делится на 7). Отсюда следует, что катушек с зелёными нитками – 3, с чёрными – 6, с белыми – 12, а с коричневыми – 27 – 21 = 6.

Ответ. 3 – с зелёными, 6 – с чёрными, 12 – с белыми, 6 – с коричневыми.

Решение1:

На языке уравнений решение выглядит таким образом:

Пусть Х-число катушек с зелеными нитками, тогда число катушек с черными нитками – 2Х, с белыми – 4Х.

20<Х+2Х+4Х<27 => 20/7

Ответ. 3 – с зелёными, 6 – с чёрными, 12 – с белыми, 6 – с коричневыми.

Комментарий

Большинство участников конкурса справились с этой задачей. Но многие потеряли несколько баллов, упустив некоторые моменты в решении. Составив уравнение вида з+2*з+4*з+к=27, они сразу же давали ответ з=3. Это не корректно, решение уравнения должно присутствовать в решении обязательно. Многие участники решали задачу перебором. Они предполагали, что количество зеленых ниток равно 1,2,3 и т.д. считали количество остальных ниток и проверяли, удовлетворяет ли полученный результат условию. Это тоже правильный подход. Но приводить ответ, утверждая, что он найдет перебором, не приводя каких-либо объяснений, не стоит. Старайтесь как можно подробней излагать свои решения. Утверждая различные факты, не доказывая их истинность, вы можете потерять баллы.

Задача №2.

Толя сильнее, чем Миша. Миша младше, чем Вова. Вова ниже, чем Толя. Толя старше, чем Вова. Вова слабее, чем Миша. Миша выше, чем Толя. Кто из ребят самый сильный, кто самый старший и кто самый высокий?

Решение

Вначале выясним, кто из ребят сильнее всех. По условию, Толя сильнее, чем Миша, а Миша сильнее Вовы. Поэтому Толя – самый сильный.

Толя старше, чем Вова, а Вова старше Миши, поэтому Толя – самый старший.

Миша выше, чем Толя, Толя выше Вовы, следовательно, Миша – самый высокий.

Ответ. Толя – самый сильный и самый старший, Миша – самый высокий.

Комментарий

Эта задача оказалась довольно простой. Об этом говорит то, что ее решили практически все. Действительно, достаточно внимательно прочитав условие, вы сразу же найдете ответы на поставленные вопросы. Были, конечно, и те, кто не на все поставленные вопросы дал правильный ответ. Думаю виной этому их невнимательность. Проверяйте полученные ответы, тем более что во многих задачах это не стоит никаких усилий.

Задача №3.

Учащихся школы разделили на 9 групп по их интересам для организации внеклассной работы. Некоторые из этих групп разделили на 9 групп для выбора места проведения внеклассной работы. Часть образованных групп снова разделили на 9 групп для предоставления возможности выбора руководителя. Могло ли образоваться в результате этих действий 105 групп?

Решение:

Что происходит, когда некоторую совокупность делят на п групп? Была одна совокупность, станет п групп, число групп увеличилось на п – 1. То же самое происходит, когда любую из полученных групп делят на п частей. Посмотрим, что происходит с общим числом групп. Была 1 совокупность, после деления её на п групп станет п групп, если ещё одну группу разделить на п групп, то число групп будет равняться п + п – 1 = 2п – 1. После деления ещё одной группы число групп станет равным 3п – 2. И т. д. Итак, если некоторая совокупность делится на п частей, некоторые из полученных частей делятся на такое же количество меньших частей и т. д, то общее число образовавшихся частей при делении на п – 1 дают в остатке 1. Число 105 при делении на 9 – 1 = 8 даёт в остатке 1. В результате проведенных действий может образоваться 105 групп. Например, следующим образом: вначале было 9 групп учащихся по их интересам для организации внеклассной работы, из них три группы разделили на 9 групп для выбора места организации внеклассной работы. Число групп стало равняться 9 – 3 + 27 = 33. 9 из этих 33-х групп разделили на 9 групп для предоставления возможности выбора руководителя. Число таких групп стало равняться 33 – 9 + 81 = 105.

Ответ. Могло.

Комментарий

К этой задаче было множество неверных решений вида:

105 не делится на 9. Ответ «нет».

9*9=81, а 9*9*9=729. Ответ «нет».

Каждый раз добавляется по 8 групп. 105 не делится на 8. Ответ «нет».

В первом случае, участники просто не учли тот факт, что при делении на 9 групп одна группа исчезает. Во втором случае участники почему-то посчитали, что на каждом шаге делятся все группы. Хотя в условии четко написано «Некоторые из этих групп разделили на 9 групп». В третьем же примере допущена совсем глупая ошибка. Участники просто не учли, что 9 групп мы имеем изначально.

И опять же, главная ошибка их всех в том, что они не попытались проверить свои ответы. Когда в задаче спрашивается «могло ли …» начните решение с попытки построить пример. Если даже построить пример у вас не получится, вы обязательно заметите какие-либо закономерности, которые помогут вам в дальнейшем. В данном случае попытавшись построить пример, вы бы точно заметили тот факт, что при каждом делении одной группы на 9, общее количество групп увеличивается на 8. Да и вообще, думаю в этой задаче построить пример смог бы каждый из вас. Надо только попробовать.

Задача №4.

Карлсон и Малыш из дома Карлсона направились в гости к другу. Карлсон преодолевает расстояние до дома друга за 30 мин., а Малыш это же расстояние – за 40 мин. Карлсон вышел через 5 мин. после выхода Малыша. Через сколько минут после своего выхода Карлсон догонит Малыша?

Решение 1:

Понятно, что предполагается, что Малыш и Карлсон движутся равномерно. Так как Карлсон преодолевает расстояние до дома друга на 10 минут быстрее Малыша, то половину этого пути Карлсон преодолеет на 5 минут быстрее. То есть через 15 минут после своего выхода Карлсон буден на середине пути, а малыш через 20 минут после своего выхода будет на середине пути. Так как малыш вышел на 5 минут раньше, то на середине пути они окажутся одновременно. Значит, через 15 минут после своего выхода Карлсон догонит малыша.

Решение 2:

Пусть v(км/ч) – скорость движения Карлсона. Так как одно и то же расстояние Карлсон преодолевает за 30 мин, а Малыш за 40 мин, то скорость малыша равна 3/4*v (км/ч). Пусть t(ч) – время после выхода Карлсона, за которое он догонит Малыша. Составим уравнение:

v*t=3/4*v*(t+1/12), левая часть уравнения это путь который пошел Карлсон до встречи с Малышом, правая часть – путь Малыша до встречи с Карлсоном (1/12=5 мин).

Как видно наше уравнение не зависит от того чему равно v (по условию v не равно 0). Если сократить обе части на v, уравнение будет иметь вид:

t=3/4*t+3/4*1/12. Получаем, что 1/4*t=1/16 значит, t=1/4(то есть 15 мин). Значит, через 15 минут после своего выхода Карлсон догонит малыша.

Комментарий

Данная задача имеет много способов решения. Я выбрал для вас именно эти два по той простой причине, что первый это самый легкий способ решения этой задачи, а второй это способ решения задачи в общем виде. То есть если числа данные в задаче не будут такими удобными (например, вместо 30 будет 33, а все остальное останется неизменным), то первым способом задача не решится. Тут на помощь придет второй способ решения, хотя коэффициенты будут совсем некрасивые.

Приведу еще один способ решения, но ну буду вдаваться в подробности. Можете попробовать сами довести его до здравого смысла, это будет полезное упражнение. Можно также как и во втором решении взять скорость Карлсона за неизвестный параметр. Далее выразить через него скорость Малыша. Посчитать разницу скоростей Карлсона и Малыша (это будет скорость, с которой Карлсон догоняет Малыша, то есть скорость сближения). Потом выразить расстояние, которое преодолел Малыш за 5 мин. И наконец, найти время, за которое преодолевается полученное расстояние, если двигаться со скоростью сближения Карлсона и Малыша. Это и будет время, за которое Карлсон догонит малыша.

Это не самый простой способ решения данной задачи. Но если сделать все достаточно аккуратно, то в итоге вы получите правильный ответ. Многие задачи имеют различные способы решения. Выбирайте тот, который вам по душе. А еще лучше, если вы приведете сразу несколько способов решения, это будет для вас только плюсом.

Задача №4.

Какое наименьшее число книг можно выдать упаковками по 5 или по 8 книг ровно двумя способами?

Решение

Очевидно, что 40 книг можно выдать двумя способами: 8 упаковок по 5 книг, или 5 упаковок по 8 книг. Убедимся, что других способов для 40 книг нет. Упаковками по 5 книг можно выдать 5, 10, 15, 20, 25, 30, 35, … книг, а упаковками по 8 книг – 8, 16, 24, 32, … книг. Легко проверяется, что сумма никаких двух чисел, взятых по одному из двух приведенных рядов, не равна 40. Поэтому ровно двумя способами можно выдать упаковками по 5 и 8 книг 40 книг.

Любое число из приведенных рядов, меньшее 40, и любая сумма двух чисел, взятых по одному из двух приведенных рядов, меньшая 40, может быть получена только одним способом. Это может быть установлено непосредственным перебором.

Для тех, кто владеет простейшими знаниями о делимости, можно предложить такое обоснование.

Предположим, что число, меньшее 40, можно двумя способами представить в виде 5а + 8b, где 0  а < 8, 0  b < 5, то есть 5а1 + 8b1 = 5а2 + 8b2. Тогда 5(а1 – а2) = 8(b1 – b2). Отсюда следует, что а1 – а2 кратно 8, а1 или а2 больше 8, получили противоречие.

Ответ. 40 книг.

Комментарий

Практически все участники конкурса дали правильный ответ в этой задаче. Но лишь некоторые из них смогли четко доказать, что полученный ответ наименьший, и что нет третьего способа выдачи 40 книг упаковками по 5 и 8. Участники, не доказавшие этих вещей, получили малое количество баллов за эту задачу. Ведь без доказательства таких моментов решение не является полным. При вопросе «найти наименьшее …» не забывайте доказывать отсутствие ответов меньших найденного вами. Это очень распространенная ошибка.


  1. 6-7 класс

Задача №1.

Тетрадь стоит 40 руб., ручка – 12 руб., а стержень к шариковой ручке – 2 руб. Ученик купил, по крайней мере, 1 тетрадь, 1 ручку и 1 стержень - всего 100 принадлежностей, израсходовав на покупку всего 400 руб. Сколько и каких принадлежностей он купил?

Решение:

Пусть было куплено а тетрадей, р ручек и с стержней. Тогда, по условию, 40а + 12р + 2с = 400 и а + р + с = 100. Выразив с из второго равенства и подставив его в первое, получим уравнение 38а + 10р = 200 или 19а + 5р = 100.

Так как а и р принимают целые положительные значения и 5р и 100 делятся на 5, то 19а делится на 5. Это возможно только при а = 5 (при других значениях а значение р будет отрицательным). При этом р = 1. Следовательно, с = 100 – 5 – 1 = 94. Итак, куплено 5 тетрадей, 1 ручку и 94 стержня.

Ответ. 5 тетрадей, 1 ручка и 94 стержня.

Комментарий

Некоторые участники конкурса дали правильный ответ к задаче, но ничего не написали о том, как его нашли. Сомневаюсь, что они получили ответ подбором, поэтому считаю, что они поступили неправильно, не оформив в работе свое решение. Я уверен, что большинство из них решали задачу именно таким способом, который приведен выше. Так как это самый очевидный подход к решению подобных задач.

Всегда оформляйте свои решения, какими бы они небыли. Даже «за уши натянутое» решение оценивается гораздо выше, чем просто ответ.

Задача №2.

В эстафете от одной команды участвовало три мотоциклиста. Один из них ехал со скоростью 60 км/ч некоторое целое число часов. Второй был в пути 3 ч, его скорость выражается целым числом км/ч. Третий проехал 120 км. Могли ли они втроём проехать всего ровно 650 км?

Решение:

Нетрудно заметить, что расстояние, которое преодолел каждый мотоциклист, кратно трём. Первый ехал со скоростью 60 км/ч целое число часов, второй был в пути 3 ч, его скорость выражается целым числом км/ч, третий проехал 120 км. Следовательно, и всё расстояние, которое проехали три мотоциклиста, должно делиться на 3. Но число 650 на 3 не делится. Поэтому это расстояние они не могли проехать.

Ответ. Не могли.

Комментарий

Не каждый сразу, прочитав условие, заметит такое противоречие, из которого сразу следует решение задачи. Здесь логично начать с того чтобы построить уравнение вроде 60*a+3*b+120=650, где a – время которое ехал первый, b – скорость с которой ехал второй. Далее попытавшись сделать в нем некоторые преобразования, чтобы упростить уравнение, вы заметите, что в левой части будет число целое, а в правой дробь.

Задача №3.

Участок прямоугольной формы огорожен 30-ю бетонными плитами длиной 1м каждая, примыкающими вплотную друг к другу. Может ли его площадь быть больше 60м2?

Решение 1:

Задача сводится к оценке площади прямоугольника, периметр которого равен 30 м и длины сторон – целые числа.

Пусть а – длина стороны прямоугольника, периметр которого равен 30 м (см. рис.). Тогда длина другой стороны равна 15 – а, а его площадь равна а(15 – а). По условию, а принимает целые значения. Составим таблицу значений выражения а(15 – а) при допустимых значениях а, то есть при а = 1, 2, 3, …, 14.

А

1

2

3

4

5

6

7

8

9

10

11

12

13

14

а(15 – а)

14

26

36

44

50

54

56

56

54

50

44

36

26

14

Пользуясь этой таблицей, можно сделать вывод, что площадь участка не может быть больше 60 м2.

Решение 2:

Докажем, что прямоугольник с фиксированным периметром имеет наибольшую площадь, когда он является квадратом. Для этого надо доказать неравенство ab<((a+b)/2)2, где a и b различные положительные числа.

((a+b)/2)2=(a2+2ab+b2)/4, умножим на 4 обе части неравенства.

4ab< a2+2ab+b2

0< a2-2ab+b2, это верно так как (a2-2ab+b2)=(a-b)2.

Таким образом, получаем, что прямоугольник с периметром 30 м имеет наибольшую площадь, когда его стороны равны по 7,5 м. 7,52=56,25. Значит его площадь не может быть больше 60 м2.

Комментарий

Вообще то, что прямоугольник с фиксированным периметром имеет наибольшую площадь, когда он является квадратом, очень часто используется в олимпиадных задачах. Очень хорошо если вы знакомы с этим фактом, а еще лучше, если вы умеете его доказывать. Конечно, перебором эта задача решается совсем не сложно. Но если в задаче будет дан периметр не 30 м, а 300 м вы же будете перебирать все возможные варианты площади.

Задача №4.

Какое наименьшее количество точек можно разместить внутри выпуклого шестиугольника, чтобы внутри каждого треугольника, вершинами которого являются вершины шестиугольника, содержалась хотя бы одна из размещённых точек?

Решение:

Нетрудно заметить, что трёх точек недостаточно. На рис. 1 изображены 4 треугольника, вершинами которых являются вершины шестиугольника. И каждый из них должен содержать точку.

Четыре точки можно выбрать внутри шестиугольника так, чтобы внутри каждого треугольника, вершинами которого являются вершины шестиугольника, содержалась хотя бы одна из размещённых точек. Например, это можно сделать так, как показано на рис. 2.

Ответ. 4.

Комментарий

Есть различные способы показать, что трех точек недостаточно (рис. 3).

А вот построить пример для четырех точек не так-то просто. Возможно, это и стало причиной того, что с этой задачей справились далеко не все. По поводу того, как построить такой пример могу сказать лишь то, что отметив все треугольники (как это показано на рис.2), можно ставить в произвольное место точку и закрашивать места, в которых точек больше быть не должно (исходя из примеров на рисунках 1 и 3). Потратив немного времени, вы все же получите искомый пример.

Задача №5.

Какое наименьшее число книг можно выдать упаковками по 5 или по 8 книг ровно тремя способами?

Решение:

Тремя способами можно выдать 80 книг упаковками по 5 и 8 книг: 810 = 80, 58 + 85 = 80, 516 = 80.

Покажем, что других способов выдачи 80 книг имеющимися упаковками не существует. Предположим, что 80 книг можно выдать а упаковками по 5 книг и с упаковками по 8 книг, то есть 5а + 8с = 80. В этом равенстве одно слагаемое 8с и сумма 80 делится на 8. Из свойств делимости целых чисел вытекает, что и второе слагаемое 5а делится на 8. Это возможно при а = 0, а = 8 и а = 16. Других значений а принимать не может, так как значение 5а превысит 80. Получили три рассмотренные способа. Других способов нет.

Пусть существуют три способа представления числа, меньшего 80, в виде 5а + 8с, то есть существует три пары чисел а1, с1; а2, с2; а3, с3 таких, что 5а1 + 8с1 = 5а2 + 8с2 =  5а3 + 8с3 < 80, где 0  аі < 10, 0  сі < 16. Можно считать, что с3 > с2 >с1. Тогда с3 < 10, с2 = с3 – 5 < 5, с1 = с2 – 5 < 0. Получили противоречие.

Ответ. 80.

Комментарий

Большинство участников конкурса дали правильный ответ в этой задаче, но лишь некоторые из них смогли доказать, что этот ответ наименьший и не существует четвертого способа выдачи 80 книг. Участники, не доказавшие этих вещей, получили малое количество баллов за эту задачу. Ведь без доказательства таких моментов решение не является полным. При вопросе «найти наименьшее …» не забывайте доказывать отсутствие ответов меньших найденного вами. Это очень распространенная ошибка.


  1. 8-9 класс

В классе 35 учеников написали по три контрольные работы по математике. В результате ни один ученик не получил оценки 2, каждый ученик получил ровно одну 3, ровно одну 4 и ровно одну 5. Узнав об этом, один ученик заметил, что по крайней мере 6 человек получили одинаковые отметки по всем трём контрольным, а другой, подумав, сказал, что таких учеников с одинаковыми отметками, наверное, будет 7. Кто из них наверняка прав?

Решение:

Разобьём класс на группы в соответствии со всевозможными наборами оценок: 3, 4, 5; 3, 5, 4; 4, 3, 5; 4, 5, 3; 5, 3, 4; 5, 4, 3 (всего 6 групп). Если в каждой из этих групп не больше 5 человек, то в классе не больше 30 человек, что противоречит условию. Следовательно, по меньшей мере в одной из этих групп не менее 6 человек. В то же время может случиться, что в каждой группе не больше 6 человек (например, в одной группе 5, а в остальных – по 6 человек), и, следовательно, утверждение второго ученика может быть неверным. Итак, в общем случае прав только первый ученик.

Ответ. Только первый.

Комментарий

Большинство участников конкурса смогли правильно ответить на вопрос задачи. Но многие из них не привели никакого решения, попросту написав ответ. Если в задании требуется оформление подробного решения, то один только ответ не принесет вам большого количества баллов. Помните это.

Задача №2.

В классе у каждого ученика есть не менее двух друзей из этого класса. Можно ли класс разбить на две группы так, чтобы у каждого ученика был в его группе друг?

Решение

Естественно считать, что в классе более трёх учеников. Разобьём класс на две группы М1 и М2 с условием: в М1 есть два друга и в М2 есть два друга. Нетрудно убедиться, что это возможно. Возьмём любого ученика х. У него есть друг у. У каждого из них есть ещё, по крайней мере, по другу. Если он общий (обозначим его z), то в группу М1 берём любого ученика из оставшихся и его одного друга, а в группу М2 – двоих из учеников х, у, z. Если у х и у есть разные друзья z1 и z2, то пары (х, z1) и (у, z2) принадлежат группам М1 и М2.

Будем рассматривать поочерёдно всех остальных членов группы М1. Если у него нет друга в группе М1, то переводим его в группу М2, там у него обязательно хотя бы один друг имеется. Если же у него есть друг в М1, то оставляем его в этой группе. После такой процедуры получим новые непустые группы М1 и М2. В группе М1 у каждого есть друг в группе. Затем такой же отбор делаем в группе М2. В его результате появятся две группы М′′1 и М′′2, в каждой из которых у каждого есть друг в его группе.

Ответ. Можно.

Комментарий

На самом деле, в задаче ничего не сказано о количестве учеников в классе. А значит, надо рассмотреть все возможные случаи. И в случае, когда в классе всего 3 ученика, выполнения условия задачи не возможно. Но в данном конкурсе отсутствие упоминания об этом не принималось за ошибку. Очень распространенной ошибкой было то, что участники решали задачу в частном случае. Например, взяв за количество учеников количество учеников своего класса. Это неправильно. И тем более неверно, когда расставляются определенные дружеские связи между рассматриваемыми учениками. Рассмотрение такого частного случая стоило всего один балл.

Задача №3.

Восьмиклассники выстроены в шеренгу. Перед каждым из них стоит семиклассник, который ниже его по росту. Докажите, что если шеренгу восьмиклассников выстроить по росту и перед ней шеренгу семиклассников выстроить по росту, то по-прежнему каждый восьмиклассник будет выше стоящего перед ним семиклассника.

Решение

Пусть А – п-й по росту семиклассник. Найдётся по крайней мере п восьмиклассников, которые выше А. Действительно, существует п семиклассников, которые не ниже А, и те восьмиклассники, которые стоят за ними при первоначальном построении, заведомо выше А.

После перестроения А стоит на п-м месте в шеренге семиклассников, из доказанного выше следует, что первые п мест в шеренге восьмиклассников займут ребята, которые выше А.

Комментарий

Неоднократно встречались работы, в которых ученики утверждали, что из условия следует, что каждый восьмиклассник выше любого семиклассника. Это не верно. Обосновывайте поставленные утверждения. Если бы ученики, допустившие подобную ошибку, попытались обосновать этот факт, они бы заметили, что это утверждение ложно.

Задача №4.

На всех гранях куба с ребром 9 см проведены полосы шириной в 1 см, симметричные относительно осей симметрии граней, как показано на рисунке. В местах пересечений полос в кубе проделаны сквозные отверстия, а вдоль полос вырезаны канавки глубиной в 1 см. Найдите объём полученного тела.

Решение:

Полученное тело состоит из 8 кубов с ребром 4 см и 12 прямоугольных параллелепипедов с высотой 1 см и квадратным основанием со стороной 3 см. Следовательно, объём тела равен 843 + 12132 = 620 см3.

Ответ. 620 см3.

Комментарий

Также, решать задачу можно и с другой стороны. Можно от объема куба с ребром 9см отнимать объемы вырезанных участков. В итоге получить тот же самый ответ. Кстати говоря, многие так и сделали. Хотя метод, приведенный в решении, более простой и наглядный.

Задача №5.

На четырёх стенах комнаты и на потолке нужно расклеить 45 снежинок так, чтобы на каждой стене была хотя бы одна снежинка, чтобы на всех стенах было разное количество снежинок и чтобы суммы количеств снежинок на противоположных стенах были равны и равнялись числу снежинок на потолке. Сколько существует различных вариантов выполнения этого задания, если различные варианты отличаются числом снежинок хотя бы на одной стене?

Решение:

Если обозначить через (m1, n1), (m2, n2) количества снежинок на противоположных стенах, а через с – количество снежинок на потолке, то будут выполняться соотношения: m1 + n1 = m2 + n2 = с.

Так как, по условию, m1 + n1 + m2 + n2 + с = 45, то суммы чисел на противоположных стенах, а также число снежинок на потолке должны равняться 15. Число 15 можно представить в виде суммы двух различных натуральных чисел семью способами:

15 = 14 + 1 = 13 + 2 = 12 + 3 = 11 + 4 = 10 + 5 = 9 + 6 = 8 + 7.

Каждые два из них определяют способ наклеивания снежинок на стены и потолок комнаты, удовлетворяющий условию. Выбрать две пары из семи можно 76 = 42 способами. Каждый из этих способов приводит к 8 вариантам наклеивания снежинок на стены и потолок комнаты. Всего будем иметь 428 = 336 вариантов выполнения задания.

Ответ. 336.

Задача оказалась довольно не простой. Не так много участников конкурса смогли ее правильно решить. Некоторые почти довели решение до конца, добравшись до 42 вариантов выбора двух пар из семи. Но не учли перестановки, в результате которых, каждый из этих 42 вариантов приводит к восьми различным способам расклейки снежинок на стенах. Хотя, в условии четко сказано, что различные варианты отличаются числом снежинок хотя бы на одной стене. Внимательно читайте условия, чтобы не совершать подобных ошибок.




Предварительный просмотр:

Error! Reference source not found.


Разбор задач третьей части заданий

  1. 4-5 класс

Задача №11.

Шестнадцать одинаковых снежинок нужно расклеить по четырём стенам комнаты так, чтобы на каждой стене была хотя бы одна снежинка, на всех стенах было разное число снежинок и суммы количеств снежинок на противоположных стенах были равны. Сколько существует различных вариантов выполнения этого задания, если различные варианты отличаются числом снежинок хотя бы на одной стене?

А. 32

Б. 24

В. 12

Г. 4.

Решение:

Так как суммы чисел снежинок на противоположных стенах равны между собой и снежинок всего 16, то всего на двух противоположных стенах 8 снежинок.

Число 8 в виде суммы двух различных натуральных чисел можно представить тремя способами: 8 = 7 + 1, 8 = 6 + 2, 8 = 5 + 3. Число различных вариантов выбора двух пар чисел, суммы которых равны 8, равно трём:

1) (7,1), (6,2); 2) (7,1), (5,3); 3) (6,2), (5,3).

Каждому такому варианту соответствует восемь различных способов украшения стен комнаты:

Следовательно, всего существует 24 различных вариантов выполнения задания.

Ответ. Б. 24.

Комментарий

В данной задаче нет ничего сложного. С ней справилось достаточно много участников. Главное учесть все варианты и аккуратно их посчитать.

Задача №12.

Квадрат разрезали прямолинейно на две части. Потом одну из частей снова таким же образом разрезали на две части. Всего сделали 50 разрезов. Какое наибольшее число вершин могут иметь многоугольники, полученные в результате этих разрезаний?

А. 54

Б. 53

В. 29

Г. 4.

Решение:

Одним прямолинейным разрезом многоугольника можно увеличить количество вершин одной из его частей не более чем на 1 (см. рис. 1 и 2). Поэтому в результате 50 разрезаний из квадрата можно получить многоугольник с числом вершин, не превосходящим 4 + 50 = 54. А получить многоугольник с 54 вершинами легко: достаточно каждый раз отрезать треугольник, у  которого только одна из вершин совпадает с вершиной многоугольника, а две стороны лежат на сторонах многоугольника (см. рис. 3).

Ответ. А. 54.

Комментарий

Большинство справилось с этой задачей. Было несколько участников, которые ответили «Г. 4.». Думаю, они сделали это, просто, не разобравшись с условием. Если условие содержит какие-то непонятные вам моменты, не надо отбрасывать задачу. Задайте свои вопросы, написав на почту, указанную на сайте. Вы обязательно получите необходимые вам ответы.

Задача №13.

Две школы соревновались в нескольких конкурсах. В каждом конкурсе за победу команде присуждали 3 очка, за ничью – 2 очка, за поражение – 1 очко. Сколькими из следующих результатов: 13:15, 19:5, 24:15, 26:18 могло закончиться соревнование между этими школами?

А. Одним

Б. Двумя

В. Тремя

Г. Четырьмя

Решение:

В каждом конкурсе команды в сумме получали 4 очка. Поэтому сумма очков, полученных командами в соревновании, должна делиться на 4. Из приведенных ответов только один – 24:15 – не удовлетворяет этому требованию.

Нетрудно убедиться, что первый и четвёртый результаты осуществимы. Например,

13:15  12:12  10:10  8:8  6:6  4:4  2:2  0:0;

26:18 23:17 20:16 17:15 14:14 12:12  10:10 8:8 6:6 4:4 2:2 0:0.

А вот результат 19:5 нельзя получить. Даже, если вторая команда все игры проиграла, первая за эти пять игр могла набрать только 15 очков. Итак, соревнование могло закончиться двумя из приведенных результатов.

Ответ. Б. Двумя.

Комментарий

Были работы, в которых участники допустили ошибку в этой задаче. Настоятельно рекомендую вам проверять свои ответы. Тем более что зачастую это совсем не требует много времени и усилий. Люди которые посчитали, что правильный ответ В или Г, не проверяли свои ответы. Иначе, не сумев построить примеры для некоторых результатов, они бы задумались о том, почему это не получается. Думаю, они смогли бы найти верное решение. И те, кто ответил «А», похоже, тоже не пытались построить примеры для предложенных результатов. Так как с этим не должно возникать никаких трудностей.

Задача №14.

У Пети 8 больших конвертов, в некоторых из них по 8 меньших конвертов, а в некоторых из меньших – по 8 совсем маленьких конвертов. Всего у него 80 конвертов. В скольких из них лежат другие конверты?

А. 7

Б. 8

В. 9

Г. 10

Решение 1:

Пусть в р больших конвертах находится по 8 меньших конвертов, то есть всего 8р таких конвертов. В части из них, обозначим их число через q, находится по 8 маленьких конвертов, таких конвертов 8q. Всего у Пети 8 + 8р + 8q конвертов. По условию, 8 + 8р + 8q = 80, или 1 + р + q = 10. Тогда р + q, то есть число конвертов, в которых лежат другие конверты, равно 9.

Ответ. В. 9.

Решение 2:

Можно сделать проще. У Пети всего 80 конвертов. Из них 8 больших. Все остальные 72 конверта лежат внутри других конвертов. Раз конверты лежащие в других конвертах упакованы по 8 штук, то конвертов с меньшими конвертами внутри 72:8=9.

Комментарий

Вообще при втором способе решения необходимо еще показать на примере, что такое возможно, но в данном случае мы имеем варианты ответов и точно знаем, что среди них один верный. Поэтому можно смело отвечать «В. 9».

Для примера можно рассмотреть задачу с аналогичным условием, но общим количеством конвертов равным не 80, а 592. Тогда решая задачу методом аналогичным решению 2, мы получим, что конвертов с другими конвертами будет (592-8):8=73. Но всего может быть не более чем 8+8*8=72 конверта с другими конвертами внутри (наибольшая сумма больших и маленьких конвертов). Поэтому такая задача не будет иметь решения.

Задача №15.

Разрезая изображённые квадраты на части по сторонам клеток, можно сложить квадрат размером 7×7 клеток. Это можно сделать несколькими способами. Какую наименьшую сумму длин разрезов можно при этом получить, если за единицу масштаба выбрать сторону клетки?

А. 6

Б. 8

В. 10

Г. 12

Решение:

На рис. 1, 2, 3 указаны три способа разрезания данных квадратов и составления квадрата 7×7.

На рис. 1 общая длина разрезов равна 8 (3 + 3 + 2).

На рис. 2 общая длина разрезов равна 8 (2 + 2 + 2 + 2).

На рис. 3 общая длина разрезов равна 9 (2 + 3 + 3 + 1). Все другие способы разрезания, например на квадратики, только увеличивают длину разрезов.

Ответ. Б. 8.


  1. 6-7 класс

Задача №11.

В первенстве округа по футболу участвует 16 команд, причём каждая команда играет со всеми остальными. За победу команде присуждается 3 очка, в случае ничьей каждая команда получает по 1 очку, за поражение – 0 очков. В первом круге все команды набрали вместе 312 очков. Сколько матчей закончилось вничью?

А. 56

Б. 54

В. 50

Г. 48

Решение:

Если игра завершается победой одной из команд, то играющие команды вместе набирают 3 очка, если ничьёй, то – два очка. Каждая из 16 команд проводит 15 матчей, в произведении 1615 = 240 каждая игра учтена дважды. Поэтому в первом круге было сыграно 240:2 = 120 матчей. Если бы все игры закончились победой одной из команд, то все команды набрали бы вместе 1203 = 360 очков. Но они фактически набрали на 360 – 312 = 48 очков меньше. Так как каждый ничейный результат уменьшает количество набранных очков на 3 – 2 = 1 очко, то 48 матчей закончились вничью.

Ответ. Г. 48.

Комментарий

Довольно простая текстовая задача, не требующая особых познаний. С ней справились практически все.

Задача №12.

В соревновании по фигурному катанию 2 команды имели одинаковое число участников. В итоге общая сумма баллов, полученных всеми участниками, равна 86. Сколько было участников в соревновании, если каждый из них получил либо 5, либо 6 баллов?

А. 14

Б. 16

В.18

Г. 20

Решение 1:

Обозначим через х и у число участников соревнований, получивших соответственно оценки 5 и 6. Тогда, по условию, 5х + 6у = 86 или 5х = 86 – 6у. Так как при допустимых значениях у правая часть последнего равенства кратна 5 только при у = 6 и у = 11, то соответственно х = 10 и х = 4. По условию, в командах одинаковое число участников. Поэтому х + у – чётное число. Этому условию удовлетворяют из приведенных значений только х = 10, у = 6. Следовательно, в соревновании было 16 участников.

Ответ. Б. 16.

Решение 2:

Так как минимальное количество баллов у каждого участника равно 5, а всего набрано 86 баллов, то количество участников не может быть больше 17 (18*5=90). Аналогично, исходя из того, что максимальное количество баллов у каждого участника равно 6 получаем, что участников не может быть меньше 15 (14*6=84). Так как в двух командах было одинаковое количество участников, то общее количество участников будет четным число больше 15, но меньше 17. Это 16.

Комментарий

Многие задачи имеют одновременно много способов решения. Если в задании требуется привести подробное решение, и вы знаете несколько способов решения этой задачи, напишите то, которое вам кажется более простым и понятным, но достаточно строгим. А еще лучше, если вы оформите несколько различных вариантов решений. Таким образом, случайно допустив ошибку в одном из них, вы сможете компенсировать ее в другом и не потерять баллы, если вдруг какое-то из них окажется неверным.

Задача №13.

Решив улучшить свою математическую подготовку, Артём начал заниматься с помощью тренажёра «Повтори математику сам». В начале сентября он выполнил первый вариант теста, а затем выполнял с недельным промежутком равносильные ему второй, третий, четвёртый и пятый варианты этого теста. Каждый правильный ответ оценивался 1 баллом. Оказалось, что за пять тестирований он набрал в сумме 50 баллов, причём при каждом следующем тестировании он набирал баллов больше, чем при предыдущем. В 5-м тестировании он набрал вдвое больше баллов, чем в первом. Сколько баллов набрал Артём в результате 2-го тестирования?

А. 10

Б. 9

В. 8

Г. 7

Решение 1:

Обозначим через с1, с2, с3, с4, с5 количества баллов, полученных Артёмом в результате, соответственно, 1-го, 2-го, 3-го, 4-го, 5-го тестирований. По условию, с1 +  с2 + с3 + с4 + с5 = 50, с1 < с2 < с3 < с4 < с5, с5 = 2с1. Из этих условий вытекает, что 50 = с1 +  с2 + с3 + с4 + 2с1 > 6c1, то есть с1 < 9, а 50 = с1 +  с2 + с3 + с4 + с5 < 5с5, то есть с5 > 10. Так как с5 = 2с1 > 10, то с1 > 5.

Если с1 = 6, то с5 = 12. Но тогда с2 + с3 + с4 = 50 – 18 = 32. Так как с4  11, с3  10, с2  9, то с2 + с3 + с4  30. Следовательно, с1 не может равняться 6.

Если с1 = 7, то с5 = 14 и с2 + с3 + с4 = 29. Учитывая, что 8  с2 < с3 < с4  13, имеем два варианта: с2 = 8, с3 = 9, с4 = 12 и  с2 = 8, с3 = 10, с4 = 11, то есть с2 может равняться только 8. Если с2  9, то с3  10, с4  11 и с2 + с3 + с4  30.

Если с1 = 8, то с5 = 16 и с2 + с3 + с4 = 26, что невозможно, так как не существует трёх различных натуральных чисел, больших 8, сумма которых равна 26.

Ответ. В. 8.

Комментарий

Данную задачу, как и многие предыдущие, можно решать перебором предложенных вариантов ответа. Может быть, такой способ кому-то и покажется легче, но не все так просто.

Решение 2:

Можно заметить, что при оценке 7 баллов за второй тест максимальное количество баллов за все тесты будет равно 6+7+10+11+12=46, что меньше 50. Это позволяет убедиться в том, что ответ Г не является верным. Также можно убедиться в том, что ответ А неверен. При оценке 10 баллов за второй тест минимальное количество баллов набранных за пять тестов будет равно 7+10+11+12+14=54. С ответом Б.9 уже сложнее. Минимум, получается, может быть 6+9+10+11+12=48 баллов, что не противоречит условию. За последний тест понятно меньше 12 баллов Артем получить не мог, если за второй у него 9 баллов. Также мы знаем, что Артем за последний тест получил четное количество баллов, так как оно вдвойне превосходит оценку первого теста. Таким образом, нам остается проверить вариант 14 баллов за пятый тест. Минимальная общая оценка в таком случае равна 7+9+10+11+14=51 балл. Получаем, что ответ Б тоже не может быть верным. Остается построить пример для ответа В. Конечно, это не обязательно, так как, отсеяв три ответа из четырех предложенных можно с уверенностью сказать, что оставшийся ответ верный, но все же построить пример будет полезно. Так мы убедимся в том, что нигде не была допущена ошибка. Итак, 7+8+9+12+14=50.

Задача №14.

Найдите площадь треугольника, изображённого на рисунке, если площадь одной клетки равна 1 см2.

А. 48

Б. 24

В. 22

Г. 18 .

Решение:

Прямоугольник MNCP, изображённый на рисунке, состоит из 6×8 = 48 клеток, площадь каждой из которых равна 1 см2. Следовательно, площадь этого прямоугольника равна 48 см2.

Площадь треугольника АВС равна разности площади прямоугольника MNCP и суммы площадей треугольников АМВ, АРС и ВNC.

Отрезок АВ делит прямоугольник МВОА пополам. Поэтому площадь треугольника АМВ равна половине площади этого прямоугольника, состоящего из 4×6 = 24 клеток, то есть равна 12 см2.

Отрезок АС делит прямоугольник AKCP пополам. Площадь треугольника АСР равна половине площади этого треугольника, то есть (2×8):2 = 8 см2.

Отрезок ВС делит прямоугольник ВNCD, пополам. Площадь треугольника ВNC равна половине площади этого треугольника, то есть (2×6):2 = 6 см2.

Тогда площадь треугольника АВС равна 48 – (12 + 8 + 6) = 22 (см2).

Ответ. В. 22 см2.

Комментарий

Многие участники дали неправильный ответ в этой задаче. Хотя данная задача имеет огромное количество различных способов решения (как впрочем, почти любая геометрическая задача). Большинство этих решений  содержат в себе факты, которые в 6-7 классах еще не преподносятся. Поэтому вам приведен именно такой способ решения. Может быть, он и не самый простой (хотя совсем не сложный), но он не требует никаких глубоких познаний своего предмета.

Задача №15.

Какими из приведенных «плиток» нельзя покрыть без наложения все клетки квадрата, кроме одной, если квадрат состоит из 25 таких же клеток, из которых составлены плитки?

Решение:

Покрытия 24-х клеток квадрата 5×5 клеток плитками А, Б, В указаны, соответственно, на рис. 1, 2, 3.

Невозможность покрытия 24-х клеток квадрата 5×5 клеток плитками вида Г можно обосновать, рассмотрев размещение плиток в трёх угловых клетках (только одна клетка может быть не закрыта!).

Ответ. Г.

Комментарий

Хотя в данном случае, построив пример покрытия для трех видов плитки, можно смело утверждать, что четвертой плиткой нельзя покрыть квадрат согласно условию. Эта задача оказалась довольно простой и с ней справились практически все участники конкурса.


  1. 8-9 класс

Задача №11.

В результате опроса учащихся класса выяснилось, что каждый мальчик дружит с 5 девочками, а каждая девочка - с 8 мальчиками. Какому из приведенных чисел может равняться число учащихся в классе?

А. 24.

Б. 25.

В. 26.

Г. 27.

Решение 1:

Пусть в классе m мальчиков и d девочек. Тогда число «дружб», с одной стороны, равно 5m, а с другой – 8d. Следовательно, 5m = 8d. Отсюда следует, что m кратно 8, а d кратно 5, то есть m может принимать значения 8, 16, 24, …, а d – 5, 10, 15, 20, …. Составляя всевозможные значения для суммы m + d, получим из приведенных в ответах только число 26 = 16 + 10. Это и есть возможное число учащихся в классе.

Ответ. В. 26.

Решение 2:

Не трудно заметить, что если в классе всего 5 девочек и 8 мальчиков и каждая пара девочка с мальчиком дружат между собой, то у каждой девочки будет 8 друзей мальчиков и у каждого мальчика будет 5 друзей девочек. В таком случае в классе всего 13 человек. Если количество учеников в классе делиться на 13 то их можно разбить на такие группы по 13 человек, в которых каждый мальчик будет иметь 5 друзей девочек, и каждая девочка будет иметь 8 друзей мальчиков. Из предложенных вариантов ответа ответ В.26 делиться на 13.

Ответ. В. 26.

Комментарий

Вторым способом решения можно здесь можно воспользоваться только потому, что среди вариантов ответа только один верный (так устроен конкурс). Ведь рассуждая таким образом, мы не проверяем наличие правильных ответов среди оставшихся.

Задача оказалась довольно простая. С ней справились практически все.

Задача №12.

Каждого пойманного леща рыболов считал за 3 рыбы, а каждых трёх ершей – за одну. В результате он насчитал 24 рыбы, и оказалось, что общее число пойманных им лещей и ершей действительно равно 24. На сколько меньше лещей поймал рыболов, чем ершей?

А. На 3

Б. На 6

В. На 9

Г. На 12

Решение 1:

Обозначим через х число пойманных лещей, тогда из условия следует, что рыболов поймал 24 – х ершей. По его подсчётам лещи составляют 3х рыб, а ерши -  рыб. Следовательно, 3х +  = 24, откуда х = 6, то есть рыболов поймал 6 лещей и 18 ершей. Лещей меньше, чем ершей на 12.

Ответ. Г. На 12.

Решение 2:

Рассмотрим задачу в общем случае, если количество рыб равно суммарному количеству ершей и лещей. Чтобы это условие выполнялось, на каждого леща должно приходиться три ерша. Таким образом, получаем, что количество ершей в три раза больше чем количество лещей. Если общее количество рыб равно х, то количество ершей 3/4*х, а количество лещей 1/4*х. Если количество рыб равно 24, то количество ершей равно 18, лещей 6. 18-6=12.

Ответ. Г. На 12.

Решение 3:

Простой способ решения здесь перебор предложенных ответов. Нетрудно заметить, что ответ не может быть нечетным. Действительно, пусть ответ равен n, где n-нечетное. Тогда количество лещей и ершей равно x+x+n, где х-количество лещей. Так как n-нечетно, то x+x+n тоже нечетно, а значит, не может равняться 24. Остается проверить ответы Б.6 и Г.12. Если n=6, то x= (24-6)/2=9. Но тогда количество рыб равно 9*3+(9+6)/3=32, а должно получиться 24. Проверив ответ Г.12. увидим, что он правильный. ((24-12)/2)*3+(((24-12)/2)+12)/3=24.

Ответ. Г. На 12.

Комментарий

Задача имеет множество путей решения. Наверное, это и послужило тому, что основная часть участников конкурса смогли правильно ответить на поставленный вопрос.

Задача №13.

Цветочная клумба, изображённая на рисунке, состоит из участка, имеющего форму прямоугольника, на котором посажены розы, и четырёх участков, имеющих форму полукругов, диаметрами которых являются стороны прямоугольника. На закрашенных на рисунке тёмным цветом участках растёт трава. Эти участки имеют форму сегментов, которые ограничены сторонами прямоугольника и описанной около него окружностью. Остальная часть клумбы засажена однолетними цветами. Площадь участка, на котором посажены розы, равна S. Чему равна площадь части клумбы, засеянной однолетними цветами?

А. 2S.

Б. S.

В. .

Г. Ответ отличен от приведенных.

Решение

Решение задачи сводится к нахождению площади фигуры, закрашенной на рис. 2, где а, b – катеты прямоугольного треугольника, с – его гипотенуза,  – его площадь. Закрашенные фигуры получаются вырезанием из полукругов, диаметрами которых являются катеты треугольника, сегментов, образованных при вписывании прямоугольного треугольника в круг. Обозначим площадь закрашенной фигуры через Sч. Тогда, пользуясь описанием фигуры, получим равенство

Sч = .

Так как по теореме Пифагора a2 + b2 = c2, то Sч = , то есть площадь закрашенной фигуры равна площади треугольника.

Фигура, изображённая на рис. 1, состоит из двух фигур, изображённых на рис. 2. Следовательно, площадь участка, засеянного однолетними цветами, равна S.

Ответ. Б. S.

Комментарий

При решении задачи используются только формулы площади окружности и прямоугольника и теорема Пифагора. В 8-9 классах эти вещи знают все. Но, к сожалению очень многие не справились с поставленной задачей.  Думаю, это связано с тем, что задача не имеет конкретных данных (числовых). Многие просто не умеют работать с параметрами, хотя на самом деле нет никакой разницы, оперировать числами или параметрами. Тем более в задачах по геометрии. Надо просто посмотреть на данные и понять, что можно их них получить, выражая через те же имеющиеся параметры.

Задача №14.

Участок прямоугольной формы ограждён 150 цельными бетонными плитами, плотно прилегающими друг к другу, каждая длиной 2,2 м. Площадь этого участка с точностью до 10  может равняться …

А. 7000 .

Б. 6900 .

В. 1700 .

Г. 330 .

Решение

Для выбора правильного ответа исследуем, какие значения может принимать площадь прямоугольника с заданным периметром 1502,2 = 330 м.

Пусть х – длина стороны в метрах, тогда длина его другой стороны равна 165 – х (м), а площадь S равна х(165 – х) (м2). Если представить это выражение в виде , то можно сделать вывод, что площадь S не превосходит  < 6889. Поэтому площадь не может равняться 7000 м2 и 6900 м2. Не может она равняться и 330 м2. Это связано с тем, что значения выражения  увеличиваются, когда значения  уменьшаются. При этом значения выражения  уменьшаются. В нашем случае наименьшее значение х равно 2,2 м. Площадь прямоугольника со сторонами 2,2 м и (165 – 2,2) м равна 2,2(165 – 2,2) = 358,16 (м2), что больше 330 м2.

Возможность ответа В, то есть площади 1700 м2 устанавливается подбором длин сторон. Если на одной стороне участка 5 пролётов, то есть её длина равна 11 м, то длина другой стороны 154 м, тогда площадь такого участка равна 15411 = 1694  1700 (м2) (с точностью до 10 м2).

Ответ. В. 1700 м2.

Комментарий

Очередная задачка из области геометрии. Не сомневаюсь в том, что многие участники конкурса решали ее простым подбором длин сторон участка. Основная часть учеников смогли получить правильный ответ. Но нашлось довольно много людей, кто посчитал ответ Г.330. правильным. Наверное, они просто невнимательно читали условие и вместо площади нашли периметр участка. Внимательно читайте условие и старайтесь всегда проверять свои ответы, дабы избежать таких глупых ошибок.

Задача №15.

Решив улучшить свою математическую подготовку, Артём начал заниматься с помощью тренажёра «Повтори математику сам». В начале сентября он выполнил первый вариант теста, а затем выполнял с недельным промежутком равносильные ему второй, третий, четвёртый и пятый варианты этого теста. Каждый правильный ответ оценивался 1 баллом. Оказалось, что за пять тестирований он набрал в сумме 53 балла, причём при каждом следующем тестировании он набирал баллов больше, чем при предыдущем. В 5-м тестировании он набрал втрое больше баллов, чем в первом. Сколько имеется вариантов распределения баллов, полученных Артёмом за 5 тестирований?

А. 11.

Б. 12

В. 13

Г. 14

Решение

Обозначим через с1, с2, с3, с4, с5 количества баллов, полученных Артёмом в результате, соответственно, 1-го, 2-го, 3-го, 4-го, 5-го тестирований. По условию, с1 +  с2 + с3 + с4 + с5 = 53, с1 < с2 < с3 < с4 < с5, с5 = 3с1. Из этих условий вытекает, что 53 = с1 +  с2 + с3 + с4 + 3с1 > 7c1, то есть с1 < 8, а 53 = с1 +  с2 + с3 + с4 + с5 < 5с5, то есть с5 > 10. Так как с5 = 3с1 > 10, то с1 > 3. Итак, 4  с1  7.

Если с1 = 4, то с5 = 12. Но тогда с2 + с3 + с4 = 53 – 16 = 37. В этом случае с2 < с3 < с4  11. Но сумма трёх натуральных чисел, не превосходящих 11, не может равняться 37. Следовательно, с1 не может равняться 4.

Если с1 = 5, то с5 = 15 и с2 + с3 + с4 = 33. Учитывая, что 6  с2 < с3 < с4  14, имеем следующие варианты:

с1

с2

с3

с4

с5

5

6

13

14

15

5

7

12

14

15

5

8

11

14

15

5

9

10

14

15

5

8

12

13

15

5

9

11

13

15

5

10

11

12

15

 Если с1 = 6, то с5 = 18 и с2 + с3 + с4 = 29. Учитывая, что 7  с2 < с3 < с4  17, получим следующие варианты:

с1

с2

с3

с4

с5

6

7

8

14

18

6

7

9

13

18

6

7

10

12

18

6

8

9

12

18

6

8

10

11

18

Если с1 = 7, то с5 = 21 и с2 + с3 + с4 = 25, что невозможно, так как не существует трёх различных натуральных чисел, больших 8, сумма которых равна 25. Итак, имеется 12 вариантов распределения баллов, полученных Артёмом за 5 тестирований.

Ответ. Б. 12.

Комментарий

Решение задачи сводится к перебору всевозможных вариантов. И допустить ошибку здесь можно только по невнимательности. Основная часть учеников смогли все правильно посчитать. Но не обошлось и без тех, кто насчитал больше или меньше чем нужно возможных вариантов. Будьте предельно внимательны при решении задач перебором. А если требуется приводить решение задачи, то старайтесь приводить перебор максимально подробно. Это никогда не бывает лишним и поможет сохранить драгоценные баллы.